Gavin Jackson

Plot hole: It makes absolutely no sense why the Terminator, who is a programmed killer, chooses not to kill anyone in this film. In Terminator 2, he didn't kill because John, who was his master, ordered him not to. In this film, we learn that John is not his master. Kate Brewster is. And she spends most of her time complaining and trying to escape from John and the Terminator. She certainly wasn't gonna bother giving the Terminator a pep talk on no killing. So it just remains a plot hole.

Gavin Jackson

Upvote valid corrections to help move entries into the corrections section.

Suggested correction: Kate Brewster told the Terminator not to kill when she reprogrammed him in the future. It's a logical order to give since its mission is to protect. It's likely John gave her that idea in the future when telling her about the terminator from T-2 (before he died of course) who he gave the same order to.

lionhead

I agree, but it is also possible that Kate programmed him not to kill anyone.

That's what I said.

lionhead

23rd Mar 2024

Road House (2024)

Trivia: The restaurant near the bookstore where Charlie works is called the Double Deuce. Double Deuce was the name of the Road House in the 1989 film version.

Gavin Jackson

Question: Was the ill-fated Rollercoaster in the film supposed to be a working one? The owner's angry reaction to its destruction suggested that it was, but if you look closely, there were gaps in the tracks and also where was the actual rollercoaster itself. This has always confused me.

Gavin Jackson

Answer: To me, in real life, it looks like a roller coaster was already set up for demolition because of its poor conditions, and the crew was able to film it for the movie. However, in-universe, the owner kept saying they were closed. It's possible they were closed to repair the roller coaster to make it work but now it's completely destroyed and beyond repair.

Bishop73

11th Feb 2024

Salem's Lot (1979)

11th Feb 2024

Starsky & Hutch (2004)

Trivia: David Soul absolutely hated this film. However, he did enjoy working with Ben Stiller and Owen Wilson.

Gavin Jackson

18th Jan 2024

Speed (1994)

Stupidity: The cops in this film really aren't much help. When Jack Traven realizes that the garbage bin drop off is just a decoy (because Howard created a hole under it) and runs out to it, not one police officer bothers to follow him to the bin or down into the subway. They just decide, "Well, our job is done, Jack can handle it himself." Talk about useless.

Gavin Jackson

Question: We learn in the film that Shazam and Wonder Woman are friends, and (Spoiler alert) she brings him back from the dead at the end. So, why didn't she help him during the final battle or give any other much-needed assistance during the film?

Gavin Jackson

Answer: The dinner scene, where Wonder Woman has the head of the Wizard, never happened. There's no indication they knew each other, let alone were friends. In the realm where superheroes actually do exist, there'd really be no reason why she, Superman, who is supposed to know them too, or any other hero wouldn't be there to help. So the only answer would be an unsatisfactory one that sounds pedantic: she didn't help because it's a Shazam movie and not a WW or JL movie. One could say that she and the other heroes were busy with fighting crime/battles in their own city or they didn't know they needed help. She only appears at the end, it seems, to restore the god realm. It's also been said they didn't think Gal Gadot would be available to shoot her cameo scene, so Wonder Woman may not have been in the film because of a scheduling conflict. But to me, if she was meant to be in the film, they would have secured her availability long before shooting.

Bishop73

Answer: Where did you get the idea that they are friends? The movie makes it pretty clear they have never met before.

Cause they are having dinner together near the beginning. And you still didn't answer why she didn't help.

Gavin Jackson

Answer: The other superheroes don't sit around waiting for someone to call. Batman has a city full of rogues' gallery: Joker, Penguin, Riddler, Mr. Freeze, Killer Croc, Catwoman. Wonder Woman also has the same. They're busy people! But they'll come if asked or if they find out another hero needs help.

Question: Was Kittridge a good guy or a bad guy? I just couldn't figure him out by the end.

Gavin Jackson

Answer: He's not a bad guy to the extent of murdering and plotting against the good guys, but he's not exactly morally upstanding either. Kittridge doesn't want to destroy the Entity like Ethan; he wants to gain control of it on behalf of the US government and is happy to deal with the White Widow or anyone else to achieve that end. His appearance on the train isn't especially nefarious; he's just the highest bidder.

Jon Sandys

Answer: He's either.

Question: Why does Jennifer at the end never seem surprised or even question Marty over why he is dressed as a cowboy (even though Marty's family does and Dave even comments on it)?

Gavin Jackson

Answer: Not sure why it's suggested that Jennifer knew about Doc and Marty's time-traveling. She immediately wakes up and tells Marty that she "Had the worst nightmare," and then in the truck, she starts saying, "That dream I had seemed so real. It was about us, and you got fired." She then asks Marty to confirm if it was a dream. Marty only had to inform her and show her the remains of the DeLorean because she still had the "You're Fired" paper in her pocket, neither of which Marty or Doc knew she had. At that point, it would've confirmed to her that she wasn't dreaming.

Answer: Jennifer was already aware of Doc and Marty's time-traveling, while his family knew nothing about it. She'd been to the future with Marty and Doc, and previously saw Doc wearing futuristic clothes. There's no reason she should be surprised, and Marty quickly updated her about everything soon after.

raywest

28th May 2023

Cobra (1986)

Trivia: Brian Thompson, who played the Night Slasher, admitted in a recent interview that during filming, he had no idea what his character was about or why he and his gang were randomly killing people. When he asked Stallone who wrote the script, he couldn't even give him an answer. (Director George Pan Cosmatos made no effort to help.) Brian was eventually forced to come up with his own explanation to assist his performance. He wrote a two-page essay and just worked from that.

Gavin Jackson

20th May 2023

Halloween (1978)

Question: Just a curiosity question, but in the scene where Michael Myers is walking around the school yard following Tommy, was Nick Castle actually wearing the mask during filming?

Gavin Jackson

Question: I'm really confused by Eddie's behaviour. Why did he suddenly decide to abandon Vincent and Carmen. And why couldn't he just take the money at the end (and maybe give it to charity if he didn't want it as Carmen suggested). Why did he unnecessarily have to turn it into an issue with Vincent?

Gavin Jackson

Answer: Because it wasn't about the money or a personal issue with Vincent. Eddie had a sudden realization about his own integrity and what he was doing (helping to rig games' outcome to skew betting odds). He wanted to win legitimately against Vincent. Earlier, he had become rather fed up with Vincent's egotistical nature and arrogance, which led to them parting ways.

raywest

Trivia: SPOILER The film marks the third time that Patrick Stewart playing Professor X has died. The first time was in X-men 3 (2006). The second time was in Logan (2017). So his death in this makes it three times.

Gavin Jackson

19th Mar 2023

Die Another Day (2002)

Question: The Bond films have traditionally hired English directors to helm all the films (much to the annoyance of many American directors like Steven Spielberg who would love to direct a Bond film). So why with this film did they decide just once to break tradition and hire a New Zealand born director instead. I'm truly baffled.

Gavin Jackson

Answer: I'm not sure about the idea that Bond films traditionally only hired English directors. Tamahori wasn't even the first New Zealander to direct a Bond film. Martin Campbell is a New Zealander who directed "GoldenEye" in 1995 (and "Casino Royale" in 2006). The first Eon Bond film, "Dr. No" was directed by Terence Young who was Irish, who directed 2 additional Bond films. Technically, Roger Spottiswoode is Canadian, but has dual citizenship. The first non-Eon Bond film, which had 5 directors credited, had 2 Americans, a German, and a Scottish director. And the 1983 film, "Never Say Never Again" was directed by Irvin Kershner who was American. And following Tamahori, there have been Swiss and American directors of Bond films (Marc Foster and Cary Joji Fukunaga).

Bishop73

Stupidity: After all the singing and dancing at the Tent Revival, Lt Elizondo yells at her fellow officers "It's a Trick, they've brainwashed the Captain, get 'em." Considering her and everyone else just witnessed the Captain float into the sky surrounded by fireworks and miraculously change into a black suit her "brainwashing" assumption is truly stupid.

Gavin Jackson

Question: I have two questions. Firstly, where did they film the scenes when Bond and Anya first encounter Jaws, it looks like the Karnak Temple to me. And secondly, why was Bond so desperate to rescue Anya at the end, considering he knew full well that Anya was planning to kill him after the mission was complete (although thankfully she changed her mind).

Gavin Jackson

Answer: First, it was indeed filmed at Karnak. Second, he rescues her because they are, for the time being, on the same side-therefore, she is an ally, and Bond won't just leave her to drown/die. I think, if he is even mindful of her promise during the heat of the moment, he would still consider it wrong to leave her behind; while the mission is on, she's a colleague in distress, and that's that. Once the mission is over, she can do what she likes, and Bond will deal with it then. (Though his surprise when she first draws the gun on him in the escape pod suggests he wasn't taking her threat seriously, so...bit of a freebie for him).

27th Sep 2022

Pinocchio (2022)

Trivia: When Pinocchio and Lampwick are going through the clock smashing factory on Pleasure island, Lampwick stands and fires a slingshot at a master clock. The face on the clock he fires at is the original Geppetto from the 1940 animated Pinocchio.

Gavin Jackson

27th Sep 2022

Pinocchio (2022)

8th Sep 2022

Gladiator (2000)

Trivia: Russell Crowe winning the Best Actor Oscar makes this the only Ridley Scott film to ever win any acting Oscars. None of his others have.

Gavin Jackson

11th Aug 2022

Grease (1978)

Question: Did Sonny and Marty become a couple at the end? You see them both dancing together during the finale but for the rest of the film, there's no suggestion that they are a couple, so I was a bit confused.

Gavin Jackson

Answer: It's hinted throughout the movie that they were dating already. The Pink Ladies are the T-Birds counterparts and therefore they date within the groups. Putzie/Jan, Doody/Frenchy, Kenickie/Rizzo, Sonny/Marty. Zuko is the only one not with a Pink Lady.

11th Aug 2022

Virtuosity (1995)

Question: What exactly is the crowd chanting at the fighting match? It's sounds like "Go Rumba" or something but I can't make it out.

Gavin Jackson

Question: When we first see Alan Grant, he and his team are excavating old dinosaur bones. Given that this is a time when Dinosaurs roam freely on earth, why is he even bothering? He could learn far more from simply studying a live dinosaur than its bones. And secondly, given that dinosaurs are alive and free, who is gonna have the slightest interest in bones that are millions of years old. I considered this a goof, but I'm opened to any explanations.

Gavin Jackson

Answer: The live dinosaurs are genetically tweaked recreations, they're not the "pure" dinosaurs of the past, which would still be of great interest. Plus just like any other archaeology or historical study, there's always more to be learned about the past, and a great many people are interested in what the past has to teach us.

Answer: Agree with the other answer but would add that while to date about 1000 species of dinosaurs have been identified, it is believed there are at least 1,000 more types that existed and are still to be discovered and studied. Only a tiny fraction of the known species were cloned by In-Gen and Biosyn, and, as noted in the other answer, they are not genetically pure. Also, there is much to learn about dinosaurs' habitat, range, species evolution and decline, mating habits, health and diet, the existing climate at the time, and so on. That would be why paleontologists like Alan Grant continue digging.

raywest

Answer: The existing answers are good. In the movie, Alan Grant actually asked, "Why do we dig?" and answered his question, "Because paleontology is science [fossil animals and plants], and science is about the truth. And there is truth in these rocks." [00:20:52].

KeyZOid

16th May 2022

Robocop 2 (1990)

Stupidity: Dr Faxx knew full well that Nuke was Cain's greatest weakness, but it never occurred to her to mention to the old man that holding a canister of it in front of him during the exhibition could have serious consequences. Not very bright.

Gavin Jackson

Other mistake: When the villains Patrick and Jack decide to go into the forest and pursue Hannah and the boy, all they take with them is their large guns and nothing else. Yet later on, they are both firing countless, endless rounds of bullets at Hannah, Ethan and the boy. Where did all the extra ammunition come from? They weren't carrying any bags and their tight suits don't seem to be holding anything.

Gavin Jackson

16th Sep 2021

Avengers: Endgame (2019)

Question: At the end when Gamora and the Good Nebula are speaking, Gamora asks Nebula what happens in the future and Nebula responds that she tried to kill her many times, but they eventually became friends. Why didn't Nebula bother to tell Gamora that Thanos killed her to get the Soul Stone. Seems odd that she left that out.

Gavin Jackson

Chosen answer: Nebula already alluded to Gamora being killed by Thanos earlier when she said something along the lines "You know what he does to you?" In this scene in question, Gamora is already disillusioned with Thanos and frees Nebula from captivity to battle him, thus it's not necessary to show the audience Nebula telling Gamora what Thanos did to her. She may have told her offscreen while they were on their way to find the evil Nebula.

Phaneron

Question: I am curious about one scene. After Bender gets his "stash" back from Brian, Andrew looks at Claire and shakes his head. Claire walks off and Andrew and Brian have some kind of whisper exchange and Brian then walks off. My interpretation of this scene is Andrew has just dumped Claire and Brian can't understand why he"s done it. If not, could someone explain this scene to me?

Gavin Jackson

Answer: It's rather simple. Andrew is trying to tell Claire and Brian not to go smoke pot with Bender, in addition to trying to hold back from his own desire to try it.

LorgSkyegon

Answer: Andrew and Claire were never a couple. They knew each other casually in the school hallway. They were upset that Bender would use Brian to hold his "stash." They thought Brian was too smart to do something that stupid. They were also upset at Bender for bring his stash to school and involving someone else in his scheme. They were starting to become friends at this point and disappointed at both of them.

7th May 2021

24 (2001)

Answer: It's never explained but my best guess would be that to assassinate a president-elect, they would hire a solid professional used to taking out high-profile targets who could remain anonymous. After the failed attempt he would likely have changed his face again and remained in the shadows till new work came along.

The_Iceman

Show generally

Question: Why is Tom Clancy credited as an executive producer on this show considering he passed away 5 years before the show went into production?

Gavin Jackson

Answer: As he is the author who created the Jack Ryan character that was adapted into a successful movie franchise, he could be credited as an executive producer for any TV or movie projects both before and after he died. It was announced in 2015 that the series would be produced for Amazon. Clancy died in late 2013, and he probably was involved in the series' earliest stages or discussions just prior to his death, and therefore would be credited posthumously. The title of TV or movie "executive producer" is fairly broad and can include one or more function, including securing financing, production oversight, creative input, script consultation, story concept, and more. Clancy's estate would likely continue to be involved under his name following his passing and receive profits and royalties.

raywest

While his estate would receive the profits, it's not automatic that Clancy would receive credit as a executive producer just because he wrote the novels. Authors like Michael Crichton, Douglas Adams haven't been credited as an executive producer after their death for use of their characters and works. Clancy's estate must be involved in the production in some way and rather than credit the estate, they credit the man.

Bishop73

Most likely his estate would be involved, through surviving family members, lawyers, etc. to act on his behalf in his name. No one said it was "automatic." It would have been a contract arrangement made while he was alive and that would continue posthumously. Whatever Michael Crichton or Douglas Adams did was a different arrangement for whatever reason they chose.

raywest

Nothing in your answer suggested anything about a contract arrangement (which if true would be the reason). You implied it was automatic. You said "as the author...he would be credited...for any...projects", but that simply is not true.

Bishop73

24th Feb 2021

The Fall Guy (1981)

Strange Bedfellows - S2-E17

Continuity mistake: When the intruder sneaks into Jody's apartment, he is wearing black leather jeans. When he and Jody are fighting, he is suddenly wearing black denim jeans. After beating Jody and tying her up, he is back in his leather jeans again and keeps them on for the rest of the episode.

Gavin Jackson

11th Feb 2021

The Fall Guy (1981)

11th Feb 2021

The Fall Guy (1981)

11th Feb 2021

The Fall Guy (1981)

P.S. I Love You - S2-E20

Trivia: When Colt finds Syringes in Cassie's bag, she explains that she is a Type 1 Diabetic which stunted her growth. In real life Dana Hill who played Cassie was indeed a Type 1 Diabetic which did indeed stunt her growth. Sadly the actress would pass away from a diabetes related illness in 1996.

Gavin Jackson

Other mistake: When Wolverine and John Wraith arrive in New Orleans, the street is jammed with people and the casino they find Gambit in is also completely full of patrons. Yet somehow Wolverine being thrown through a wall, his loud shouting, his slicing a fire-ladder to pieces and Gambits massive shock thrust somehow draws absolutely no attention from anyone except the main characters.

Gavin Jackson

13th Oct 2020

The Frisco Kid (1979)

Question: When Avram is departing on his horse after meeting Tommy, Tommy asks him if he speaks any Mexican. Abram, who doesn't, is puzzled by the question and asks why...to which Tommy responds "Just curious." I've always assumed that Tommy was mocking him cause he was unknowingly riding south and headed for Mexico instead of West towards San Francisco. Am I right?

Gavin Jackson

Answer: Right on the nose.

9th Sep 2020

Cobra Kai (2018)

Season 2 generally

Question: Johnny punishes his Cobra Kai class after learning that certain members trashed Daniel's Dojo to try to get the culprits to confess (with no luck). Why didn't he just send them all to Daniel's Dojo to clean up the mess and fix it up? That would have been the more honourable thing to do.

Gavin Jackson

Answer: He was trying to flesh out the actual culprits but, as you pointed out, he wasn't having any luck. But if he hadn't received the phone call, he might have sent the entire class over to clean up. However, in the same episode, check Robby and Demitri's reactions to the former Cobra Kai students joining Daniel's school. Tensions were high between the two dojos and Daniel's anger at Johnny's dojo might have convinced Johnny that the current students would not be welcome there (risking a fight which both Johnny and Daniel wanted to avoid), so he might have been reluctant to send anyone even if it was the honorable thing to do. Also, Johnny mentions in an earlier episode that there is a difference between mercy and honor. He wants to make sure the real culprits get punished properly. Once he discovered who did it, the innocent would be off the hook and then the real punishment would begin for the guilty. So, in closing, there could be any number of reasons why he didn't send the entire class to clean up Miyagi-Do.

dewinela

31st Aug 2020

Runaway Train (1985)

Trivia: In the original script Manny was supposed to be a convicted killer but writer Edward Bunker changed it to a safe-cracker because he didn't feel the other prisoners would respect a killer. Curiously this makes Warden Rankin's bitter and homicidal hatred towards Manny (which included welding him into his cell for 2 years) rather questionable as safe cracking isn't remotely as serious as murder.

Gavin Jackson

24th Aug 2020

Alcatraz (2012)

Trivia: This show pays many homages to the 1968 film Bullitt which was also set in San Francisco. Firstly officer Rebecca Madsen's car is the exact same make, colour, and model Mustang that Steve McQueen drove in Bullitt. Also the final episode (Tommy Madsen) features a car chase which recreates many shots from the 1968 film. It even includes the green Beetle Volkswagen coming down the hill as seen in the movie.

Gavin Jackson

24th Aug 2020

Alcatraz (2012)

Garrett Stillman - S1-E12

Visible crew/equipment: When Rebecca Madsen jumps in her car to pursue the armored truck, as she does a U-turn and takes off at high speed, you can see all the people standing on the side watching the filming. Some even have their phone cameras out videoing it. (00:29:30)

Gavin Jackson

18th Jul 2020

The Lost Boys (1987)

Trivia: When David is grabbing Sam as he is escaping from the cave, he looks at his burnt hand and a tear comes out of his eye. The reason for this was Kiefer Sutherland had been wearing his vampire eye contact far too long and when he looked at his burning hand, the extreme light caused his eye to water up. This has been confirmed by Kiefer himself.

Gavin Jackson

18th Jul 2020

Alien (1979)

Question: I recently saw this film at a local cinema and I noticed that there was a scene missing from when I first saw this film as a kid. In the aforementioned scene, Dallas is on the main computer (Mother) trying to get information about how to destroy the Alien. The computer just keeps responding with "Can not compute." He finally asks "What are my chances" and still gets the same response. I was wondering if anyone remembered this scene and knows why its been removed?

Gavin Jackson

Answer: If I'm not mistaken, the scene you're talking about (where Dallas consults Mother before going into the vent) was removed from the Director's Cut version of the film (which did get its own theatrical release in 2003). Perhaps that's the version they showed. I couldn't find the scene in its entirety, but is this what you're referring to? https://m.youtube.com/watch?v=OaoQES6C9ok.

Bishop73

Yes that was the scene. It was longer of course.

Gavin Jackson

30th Jun 2020

St. Elmo's Fire (1985)

Trivia: Ally Sheedy was mortified after learning that she had to perform a sex scene as she presumed that it would be offscreen and simply implied. Director Joel Schumacher convinced her to do it by agreeing to only shoot her and Andrew McCarthy from the neck up. Also the scene where she accidentally pushes the shower screen down was unscripted and McCarthy's laughing reaction was genuine.

Gavin Jackson

30th Jun 2020

Con Air (1997)

Stupidity: For a brilliant criminal mastermind, Cyrus makes so many idiotic mistakes. (1) He leaves his entire escape plan hidden in the wall for the guards to find later instead of trying to dispose or destroy them (2) He makes the fatal mistake of trusting Santiago completely and never suspecting a double-cross. At the very least, he should have checked all of Lerner airfield just in case the plane was hiding (which it was). (3) He never notices or even questions Billy Bedlam's sudden absence until he sees his body. Odd considering he notice Santiago's absence immediately. And (4) When escaping on the firetruck, he (pointlessly) chooses to ride on the ladder instead of the front seat where he stands out, and Poe and Larkin both spot him. No surprise his whole plan failed.

Gavin Jackson

30th Jun 2020

The Fall Guy (1981)

Answer: Its current location is right back in the Australian Outback in the city of Broken Hill, New South Wales.

raywest

7th May 2020

Young Guns 2 (1990)

Question: What happened to Doc's girlfriend from the first film? She risked her life to be with him at the end of Young Guns, so its a little odd that there's no mention of her in this film.

Gavin Jackson

Answer: Doc says in the movie he is married with kids. So you have to assume he married her.

Stupidity: John Candy was being dragged by the speedboat because he was holding the bar when the boat took off. It simply never occurred to him that letting go off the bar would have solved the problem (Of course then you wouldn't have had the speedboat scene at all but it's still pretty dumb).

Gavin Jackson

Upvote valid corrections to help move entries into the corrections section.

Suggested correction: Characters doing stupid things doesn't constitute a stupidity entry. That was part of the joke, that Chet spent all the time telling his son to remember to let go of the rope if something goes wrong, but then forgets his own advice in the heat of the moment. People do stupid things in real life all the time.

Bishop73

Well how does this not count as a Stupidity then? You just said it was and there was no need for him to stay holding onto the rope.

Gavin Jackson

Stupidity is basically a minor plot hole, something small that doesn't rise to the level of an plot hole entry. Characters are still allowed to do stupid things though if it's not a plot hole (otherwise everything Lloyd and Harry do in all the Dumb and Dumber movies would be stupidity entries).

Bishop73

10th Apr 2020

F/X (1986)

Stupidity: After the Defranco "assassination" Rollie learns he is a target and survives two attempts on his life. But instead of leaving town or finding somewhere else to hide for the night, he spends the night in his own apartment (even though he was well aware the killers knew where he lived). As a result, his girlfriend is killed and he barely escapes alive.

Gavin Jackson

5th Apr 2020

Footloose (1984)

Trivia: On the DVD commentary, Kevin Bacon stated that at the time of release that despite the studio's insistence, he refused to go around telling reporters that he did all his own dancing in the film (he was mostly doubled) after the backlash that Jennifer Beals copped after Flashdance where she was forced by the studio to say she did all her own dancing despite only doing close-ups.

Gavin Jackson

31st Mar 2020

The Invisible Man (2020)

Audio problem: When the invisible man slaps Sydney (making Cecilia look guilty), the sound of a bare hand slap is heard. But the invisible man was wearing gloves as part of his suit, so the sound would have been duller and blunt.

Gavin Jackson

17th Mar 2020

Grease (1978)

Question: During the car race, we see Cha-cha say "Cmon babe." Is she referring to Danny or the Scorpions leader? (Remember that Danny wasn't originally supposed to be the driver).

Gavin Jackson

Chosen answer: She's referring to Leo, the leader of the Scorpions.

Super Grover

Answer: I think she's secretly meaning Danny because at the dance she calls him "Zuko Baby" and then at the race she says "c'mon baby"

12th Feb 2020

Doctor Dolittle (1967)

Revealing mistake: When Dr Dolittle is talking to the parrot and it raises its left leg, you can see a piece of string running past Dolittle's head used to raise its leg.

Gavin Jackson

29th Dec 2019

Bad Boys for Life (2020)

Trivia: In the trailer Martin Lawrence gets out of the passenger side of Will Smith's valuable car and his door gets damaged hitting a fire hydrant. This is a reference to an actual behind-the-scenes moment in Bad Boys 2 when Lawrence did the same thing, hitting a concrete barrier whilst exiting the expensive Ferrari used in the film. It infuriated Michael Bay and Lawrence was highly apologetic.

Gavin Jackson

Question: If Rey was powerful enough to realise that Chewbacca was still alive on another spaceship (remember that they were in space and the ship was probably a far distance away), why did she have no clue that he wasn't on the cargo ship right in front of her (the one her and Kylo accidentally blew up)?

Gavin Jackson

Answer: At this point Rey had just been involved in a test of power versus Kylo Ren - she was probably far too stressed at the time so couldn't determine whether Chewbacca was onboard or not.

True. However she may have been more baffled by the blue lightning strike that she just cast.

Question: This has always baffled me. When Ryan is talking to Jeffrey Pelt and Pelt asks him what he's supposed to do if Ramius is in fact trying to defect, Ryan responds that that they should grab the sub and Pelt responds that it is a billion dollar sub and the Russians are "gonna want it back." But Greer when first telling Ryan the news of the Red October's theft said that the Russians wanted the US's help to sink her. In other words the Russians wanted the Red October destroyed, not returned. And Pelt's counter argument made little sense (as grabbing the sub was indeed an option). Was this just poor writing or what?

Gavin Jackson

Answer: Actually it makes perfect sense. The Russians do not want the sub falling into the American's hands, period. They want to retrieve it themselves or sink it. While it would be a great financial loss to destroy it, it would be much more damaging to have the Americans capture it, because they would then be able to reverse engineer the new design.

15th Nov 2019

Titanic (1997)

Trivia: The scene at the end when everyone is running and the priest is saying "As I walk through the valley of death" to which an annoyed Jack responds "Could you walk a little faster" was taken from 1969's Paint Your Wagon where Lee Marvin and the preacher have the exact same exchange when fleeing the cave collapse.

Gavin Jackson

7th Nov 2019

Doctor Sleep (2019)

Trivia: When we see Abra's house, the number on the front is 1980. That was the same year the original Shining was released.

Gavin Jackson

Trivia: SPOILER! John Connor's death scene during the opening marks the first time since the original film that Arnold as the Terminator has killed a human.

Gavin Jackson

26th Sep 2019

Child's Play (2019)

26th Sep 2019

Avengers: Endgame (2019)

Question: Where was Gamora during the funeral scene and when the Guardians leave at the end. Was she just hiding on earth or something?

Gavin Jackson

Answer: This version of Gamora had abandoned Thanos, but she is also not a member of the Guardians of the Galaxy, so she flees to parts unknown. There's an alternate version of the scene when Tony dies showing all the heroes bowing to him, with Gamora looking back at Tony momentarily before heading off all by herself.

Phaneron

15th May 2019

Kidnap (2017)

Stupidity: When sneaking into the hillbilly's house at the end to find her son, Karla for some unknown reason places the loaded sawn-off shotgun (which she stole from the hubby) on the kitchen table and replaces it with a knife. As a result the hillbilly wife finds it and tries to use it on her later.

Gavin Jackson

11th May 2019

Avengers: Endgame (2019)

Answer: This is right out of the comics. When Steve Rogers retires as Captain America in All-New Captain America #1, he passes the mantle to Sam Wilson.

wizard_of_gore

Answer: Steve Rogers didn't have super strength when he was chosen to be Cap. His demeanor, personality, and selflessness earned him the mantle. The abilities came with it.

DetectiveGadget85

His character earned him the slot in the super soldier program - his abilities are a direct result of the now-unavailable super soldier serum. Falcon is highly skilled and trained, doesn't mean he can't do a good job as a "new" Captain America with a different skillset, but he won't have the same strength and speed.

Answer: As shown throughout the events of the "Captain America" and "Avengers" sequels, Cap and Sam are very much kindred spirits with a great deal in common. Cap thus saw him as the perfect person to pass the mantle onto.

TedStixon

4th May 2019

Speed (1994)

Trivia: In the original script. Jack's partner Harry would turn out to be the mastermind behind the bus bomb (with Dennis Hopper simply being his accomplice). When Ed Harris, who was originally cast as Harry, left and Jeff Daniels got the role, producers felt that Daniels was too nice and likeable to be truly convincing as a villain and changed it to just make Hopper the sole baddie.

Gavin Jackson

15th Mar 2019

Stargate (1994)

Revealing mistake: On the first night at the slaves' fortress, Daniel is eating the "chicken" and telling them how good it is. After that they take him away to give him a wash. As they walk off, they pass the small fire the slaves have made. If you look at the fire closely, there is a metal circle at the bottom and the fire is coming out of seven holes in it, which means that it is a prop gas fire.

Gavin Jackson

10th Mar 2019

Stargate (1994)

Question: When Jackson and O'Neill send the nuclear bomb up to Ra's spaceship, why didn't he simply send it down again? Sure it wouldn't have made it back to ground before detonation, but at least his ship may have been spared.

Gavin Jackson

Chosen answer: When Ra saw the bomb, he realised that it was a few seconds away from detonation. There wouldn't have been enough time to send it back.

9th Jan 2019

Mad Max 2 (1981)

Trivia: After Max reloads his gun, the gyro captain asks "How do I know that one's not a dud" to which Max responds "Let's find out." As they walk to the Gyro-copter and the captain is ranting on, Mel Gibson tries to holster his shotgun but fails. He quickly tries again with no luck and finally gives up and just carries the gun in frustration.

Gavin Jackson

28th Dec 2018

Commando (1985)

Question: The villain's whole plan was to force John Matrix to kill some foreign president by holding his daughter hostage. So why at the beginning were they killing members of his unit? That seemed rather pointless to me.

Gavin Jackson

Answer: They didn't know where Matrix lived. They were killing Matrix's old unit because they knew if they did General Kirby would make physical contact with Matrix to warn him. They simply followed Kirby to Matrix's home.

BaconIsMyBFF

Answer: To flush him out into the open. By killing his unit it will make it more likely he will want revenge and come out into the open making him a target.

Ssiscool

13th Dec 2018

Halloween (2018)

Question: Does anyone know which scenes Nick Castle is playing Michael Myers? I know he is Myers when Laurie tries to shoot him through the window and the after credits breathing was his. But is it him any other times?

Gavin Jackson

Answer: You didn't mention the opening scene; I'm sure you know it's him up till he gets his mask back, right?

No, it's not. That's James Jude Courtney playing Michael in the unmasked scenes.

dewinela

Answer: Based on multiple things that I've heard, those are the only two times Nick Castle is on screen. He was mostly just there to help James Jude Courtney get the mannerisms down.

hbkedge3

I had read in Halloweenmovies.com and other Michael Myers Instagram accounts that I follow, that was Nick Castle in the closet when the baby sitter Vicky opened the door.

3rd Dec 2018

Gremlins (1984)

Question: If Gizmo is not allowed to come in contact with water, then how is he so clean? Since they can't wash him, wouldn't he be smelly and covered in grime?

Gavin Jackson

Chosen answer: In the original script it was mentioned that mogwais could not reproduce if the water was below 25° (which is why the snow doesn't cause them to multiply). So it's feasible they could be washed with ice water. In addition, there are other methods of cleaning that don't require water (e.g. dry cleaning).

Bishop73

Answer: There's a possibility that Mr. Wing, Gizmo's first caretaker, used dry shampoo to keep him clean.

Answer: Google "dust bath" it's something a number of small animals do to keep clean in the absence of water (pet animals too, like chinchillas).

dizzyd

It's Only Rock and Roll - S3-E11

Stupidity: When the gangsters show up at the house Pam and Dak Hampton are hiding in, Pam is captured. Dak then attempts to flee by running out the front door. What a dumb move. No surprise that he got captured as well.

Gavin Jackson

Plague - S2-E8

Revealing mistake: When Bill is pushed out of the helicopter and Ralph rescues him in mid-air, its obvious in some shots that it is two sky-divers dressed as Ralph and Bill. Firstly you can see the parachute back-pack bulges on both men. Also the wind is blowing up on Ralph even though he isn't meant to be descending.

Gavin Jackson

26th Oct 2018

Rocky IV (1985)

Question: During the press conferences, Is there any reason why Drago remained quiet? My best guesses are a) He's shy during Q and A's, b) Just a pride thing, or c) he's simply been ordered by his superiors not to speak. Anyone know for sure?

Gavin Jackson

Chosen answer: Being Russian, English was not his first language and it may have been felt that he could not articulate himself well enough to where he wouldn't be misunderstood or misquoted.

raywest

Answer: To make him appear more menacing and machine like.

The_Iceman

Answer: There are many theories and claims, but there seem to be no definitive answer. My take is that Drago was shy and pretty much a puppet of his wife and nation, both obsessed with creating a "stone cold iron machine" in where a human personality is only in the way This theory is supported by the fact that when Rocky starts winning over Drago's respect, Drago (instead of his seemingly shy and yet cold demeanor) starts shouting something like "I AM NOT FIGHTING FOR ANY OF YOU ANYMORE! THIS FIGHT IS FOR ME! I AM FIGHTING FOR ME!" Ignoring the boos of the crowd and garnering a slight smile from Rocky Drago seemingly regains a bit of himself, his own identity during the fight of his life.

24th Oct 2018

Trainspotting (1996)

Chosen answer: Renton is wet to recall the toilet diving scene. See "The Story Behind the Groundbreaking Trainspotting Poster" on the Vice website for details.

Question: Why did Qi'ra choose to kill Dryden Vos and spare Han? Was it because she loved Han, or simply because she decided that she would rather work for Darth Maul instead?

Gavin Jackson

Chosen answer: Qi'ra's motivations are purposefully vague. It could be she killed Vos because she loved Han. It could be a simple power play, as she did not like being a slave to Vos. It could be a combination of both: she was able to save Han and also gain a position of power by killing Vos. We'll likely never know the answer as a follow up to Solo seems unlikely at this point.

BaconIsMyBFF

Question: In the US, can you really be taken to a police station for (presumably) making a phony phone call? To me it seems like a waste of resources since it is probably a common occurrence. Also Jeannie would have stuck to her story that it was true...as break-ins do happen all the time.

Gavin Jackson

Answer: In the United States, phoning in a false report to police can be treated as either a misdemeanor or felony offense (depending on jurisdiction). A false report is considered obstructing law enforcement, which is pretty serious. A misdemeanor offense can result in jail time of a year or less, while a felony offense can result in more than a year's jail time and heavy fines. The important factor that makes it a crime is knowingly trying to mislead the police, not accidentally reporting false information. In "Ferris Beuller's Day Off," Jean was not trying to mislead the police; but, lacking evidence, the police assumed she was knowingly phoning in a false report, so they arrested her (probably on a misdemeanor).

Charles Austin Miller

Answer: Illinois law makes making a false report on 911 a form of disorderly conduct.

LorgSkyegon

Answer: I think the main thing was Jeanie wasn't in school and was probably taken in for truancy as well as the supposedly phony call. But it's unlikely that they would have taken her to the police station if she was picked up at home.

Bishop73

3rd Sep 2018

True Lies (1994)

Question: Was this film a box office hit, did it flop or simply break even? I have heard nothing about it being a huge money-maker. And of all James Cameron's films, it seems to have gotten the least mention (the controversial strip tease scene might be one reason). So I was wondering if someone could clarify it once and for all?

Gavin Jackson

Chosen answer: True Lies did well, earning about $379m worldwide from a $100m+ budget, the first movie to have a production budget that high. It was the third highest grossing movie of 1994.

Jon Sandys

Answer: There was supposed to be a sequel but after 9/11. Middle East Terrorism seemed in poor taste to James Cameron.

Question: How did Claire get into the closet where Bender was? Vernon locked the door when he left and Bender had to crawl through the ceiling to get out and back in. So how did Claire simply open the door?

Gavin Jackson

Chosen answer: The key may have been left in the door knob, Bender wouldn't have known that so he returned the same way he left. Real question is how did Claire know which closet he was locked in?

Answer: Perhaps she picked the lock.

Alan Keddie

Claire closed the door once she entered. Which means if she had picked it, then she would have locked herself in. I highly doubt she wanted to perform the ceiling crawl.

Gavin Jackson

Answer: Claire didn't have to pick a lock and the door doesn't lock automatically! It's plain to see: The storeroom door is closed behind Vernon and Bender. As Vernon leaves, he uses the doorhandle and then fumbles around on the outside, "locking" the door without a key. It very much looks as if Paul Gleason acts that there's a working lock. Inattentive viewers may then mistake the door from the next shot for that of the storeroom, but it states: Faculty Lounge. As for "how did she know"? *sigh* How about something so boring that wasn't worth filming it: When Bender returned, they asked what happened and he simply told them! The real question is: How scared must he have been after Vernon's bullying to crawl back through the ceiling (risking another fall) when Claire obviously managed to sneak out and meet him?

17th May 2018

Deadpool 2 (2018)

Trivia: According to the DVD commentary, director Stephen Elliot wanted the drag queens to scale Ayers Rock (Uluru) at the end. But the park owner refused, claiming that he thought the film sounded dumb (and probably because climbing Ayers Rock is generally dissuaded). Elliot wanted to beat him up badly but didn't. In the end, he settled for Kings Canyon for the finale.

Gavin Jackson

Answer: Think of any manufacturing process. Samples of new products are frequently created and then immediately destroyed. Also, the new replicant would require processing, training, etc. It was simpler for him to just dispose of the test.

Answer: He was being violently petulant at the moment, angry that he couldn't create and control the birth that he just learned occurred with older-model replicants and seeing his new creation as "flawed" by design. Pretty villainous, he cares nothing for the replicants.

Erik M.

7th Apr 2018

Hogan's Heroes (1965)

31st Mar 2018

Fletch Lives (1989)

4th Mar 2018

Shoot to Kill (1988)

Stupidity: When Jonathan is chasing the truck to try and save Sarah, he could have removed his heavy backpack and run faster...but instead just leaves it on. No wonder he wasn't fast enough.

Gavin Jackson

Chosen answer: For two reasons, one to be polite and not act suspiciously, if he acted nervous or uncomfortable, the sheriff would be wary of him. Second, so the sheriff would leave quickly in case he had no cash or wanted to pay with a credit card. There were no slide cards back in the 1970's.

Question: In the first film, Frank took on all the world leaders and proved himself to be a skilled fighter. So why is his fight scene in this film with Hector Savage so awkward and shows Frank as being a clueless fighter?

Gavin Jackson

Chosen answer: It was also stated in the movie that Savage was a professional fighter. Therefore Frank would be no match for him.

lartaker1975

Answer: Rule of funny.

Answer: Just because he beat up a group of people doesn't mean he can beat anybody. Hector may have been stronger whereas the leaders may have been a bit weaker than him.

15th Jan 2018

Mad Max (1979)

Trivia: In a Q&A session, Steve Bisley stated that he came up with the idea of Goose laughing after he crashes into the car to show the character's reckless and crazy nature. On the day of filming, it was hot and Bisley was hardly feeling cheerful. So to make him laugh, Mel Gibson got on top of the car bisley was under and started dancing and exposing his genitals. That got Bisley laughing.

Gavin Jackson

The School Picture - S4-E30

Deliberate mistake: Although it was done for plot purposes, no school photographer would only take just one shot of a class photo. They generally take up to 15 to 20, as it is difficult to get everyone to look at the camera and smile at once (and hopefully not pull silly faces).

Gavin Jackson

18th Dec 2017

Spies Like Us (1985)

Other mistake: When Chevy chase is being filmed cheating on the exam, all the video cameras are on the sidewalls near the roof. But later when they view the footage, the last shot of Chevy copying the answers is from the floor level where no camera was seen in any previous shots. Also what was a floor level camera going to film anyway...everyone's feet?

Gavin Jackson

Question: In Empire Strikes Back and Return of the Jedi, Luke's replacement hand looks like a normal human one (though it was mechanical). In this film he has a more obviously robotic hand. Other than to remind today's audiences that he previously lost his hand, was there any point to this change?

Gavin Jackson

Answer: The outer skin could have been destroyed when Ben attacked Luke and ransacked the temple. We see Luke's robotic hand rise out of the rubble. Perhaps it had skin before that, and Luke never fixed it as a reminder of his "failure."

Answer: At the beginning of Return of the Jedi Luke gets shot in the robotic hand and just wears a glove over it for the whole movie even though he could have had the skin fixed. Apparently in the many years between the movies he never bothered having any damage to the face flesh and skin fixed.

Answer: After thirty plus years, it could just be the natural degradation of the organic flesh covering the robotic parts.

Answer: He likely downgraded to a simpler, easier to maintain version at some point. Presumably before going off to the island.

The robotic hand Luke has in The Last Jedi has the scorch mark he got from a blaster from Return of the Jedi. It's the same hand, just without the artificial skin covering.

Bishop73

Stupidity: When Snake is surfing up to Eddie's car, all Eddie had to do was hit the brakes and Snake would have helplessly surfed right past him. But instead he accelerated, idiotically.

Gavin Jackson

28th Nov 2017

The Warriors (1979)

Question: Instead of trying to murder them, why didn't the Lizzies simply contact the Riffs and tell them they had 3 of the warriors and to come get them? They had plenty of time.

Gavin Jackson

Answer: The Riffs had put out a hit on the Warriors via the radio DJ, meaning that it didn't matter if the Warriors were taken alive or not; the Lizzies probably figured it would be easier simply to kill the Warriors and collect the reward.

zendaddy621

25th Nov 2017

The Warriors (1979)

Trivia: When Cochise, Rembrandt and Vermin are fleeing the cops at the train station, they run past a bearded man with a blue sweater and flat cap who comforts his girlfriend. This man is director Walter Hill in a cameo.

Gavin Jackson

19th Nov 2017

Detroit (2017)

Question: Why didn't the guys being interrogated in the hotel just tell the cops about the toy gun that was used? 4 of them knew that it was that gun that was shot out the window and another 4 knew that the gun existed, but they all choose not to mention it. Any reason?

Gavin Jackson

Chosen answer: I think at 1st they didn't want to answer as they don't talk to police, but as it went on their lie just got bigger and the hole they were digging got so big they could not get out. They also figured the police would not believe it was fake and they would be in loads of trouble if they admitted there was any kind of gun. They call it a toy gun, but it is not really a toy as it's a starter gun.

Adam Busuttil

14th Nov 2017

Wall Street (1987)

Question: During the meeting, what made Carl Fox realise that Gordon Gecko was dishonest about his intentions? And after Bud threw Darien out of the apartment, chances are she would have told Gordon and said why. What with this and the confrontation he had with Bud earlier, wouldn't Gordon have suspected that Bud would turn against him?

Gavin Jackson

3rd Nov 2017

Mad Max (1979)

Other mistake: When Jessie realises that her baby is missing, she runs out into the yard, stops and does a slow 360 turn before looking forward again. She then hears the Toecutter's voice, looks to her right and sees all the knightriders, including one holding her baby. So somehow in that slow 360 turn she did previously, she somehow managed not to see 15 knightriders standing close to her.

Gavin Jackson

Stupidity: When the duke approaches the wall at the end, Snake dives on him and briefly knocks him out. Snake could have easily grabbed his gun back and left the duke unarmed (or even just killed the duke there and then), but just gets up and runs, leaving it with him.

Gavin Jackson

1st Nov 2017

Bronco Billy (1980)

Plot hole: Antoinette Lilly makes headlines as being murdered and her husband is charged and sent to a criminal asylum...despite the fact that no body has been found and no one has even bothered to go to the hotel or town she was last at to investigate whether or not she's alive or dead. Not believable.

Gavin Jackson

Upvote valid corrections to help move entries into the corrections section.

Suggested correction: John was framed by Antoinette's step-mother (who stood to gain Antoinette's inheritance). People have been arrested for murder without a body being discovered first though. Additionally, we find out that the lawyer who helped framed John also paid him to confess to the crime and admitted to being mentally unstable. So nobody was even trying to look for Antoinette.

Bishop73

10th Oct 2017

Mad Max 2 (1981)

Factual error: There's no way the gyro helicopter could carry 2 men and four jerrycans of petrol across a desert. It is only built for one person and would never lift off. And where would they put the second person and petrol anyway?

Gavin Jackson

10th Oct 2017

Mad Max 2 (1981)

Factual error: It's impossible for the Gyro captain's helicopter to land in the compound the way it did. Those type of helicopters can't hover like normal ones and have to land on runways due to their speed. It can't just land on the ground directly.

Gavin Jackson

10th Oct 2017

Mad Max 2 (1981)

Continuity mistake: When Max leaves the compound and is being pursued by Wez and Toadie, the ground is flat as far as the eye can see. When Max's car begins to roll, the ground is still flat. Suddenly Max winds up in a deep terrain which came from nowhere and wasn't seen in any previous shots.

Gavin Jackson

10th Oct 2017

Mad Max 2 (1981)

Other mistake: Max had already driven his car a distance along the highway before he crashed. When the Gyro captain spots the smoke the next morning, it's coming from behind the hill which is way too close and suggests that Max crashed right after passing the knight riders.

Gavin Jackson

Question: Could someone please explain to me what I consider two of the biggest plot holes in this film. Firstly, how was Snake going to get his glider back in the air? If it had jets or thrusters on it, that would make sense, but when it first took off, it was being towed by a plane which means it is just a standard glider (Plissken does admit to Hauk that it will be an issue, but that explains little). Also given that it is a one-seat glider, how was he going to bring back the president on it? In fact, how were Brain, Maggie and the President (in their attempted double-cross) all going to squeeze onboard?

Gavin Jackson

Answer: When Snake is told how he getting in the other guys said he can take off from a free fall.

Chosen answer: You are correct in citing these problems as plot holes. Writer-director John Carpenter is notorious for glaring plot holes in his films over the decades, inasmuch as Carpenter crafts his films for shock effect, rather than continuity. Other examples of Carpenter's plot holes can be found in the original "Halloween" (for instance, how did the killer Michael Myers, who had been confined in a mental institution since he was a child, learn to drive an automobile like a stunt driver his first time behind the wheel?), and in "The Thing" (why is the Thing discovered frozen in ice only about 4 feet below the modern surface while its spaceship is buried ten yards deep in 100,000-year-old ice?), as well as in "Prince of Darkness," "Vampires" and other Carpenter films. One explanation is that Carpenter tends to edit-out slower, extraneous scenes that more fully explain the plot, in order to maintain a fast-paced storyline.

Charles Austin Miller

I would assume he would push free fall and build up enough speed to fly, and then pull up, the WTC was the largest building in NY at the time anyway.

22nd Sep 2017

Beverly Hills Cop (1984)

Chosen answer: Much of Axel Foley's lines were improvised, including this scene, according to the commentary. But many other actors improvised their lines too, most notably Bronson Pinchot. The script was originally written with the intent of Sylvester Stallone playing Axel and as such, the issue of his race wasn't scripted and the director tried to avoid mentioning race in the film. It was Eddie Murphy that would bring it up in improvised lines.

Bishop73

14th Sep 2017

Ghost Story (1981)

13th Aug 2017

Snowden (2016)

Question: When Edward Snowden obtains the MicroSD card after downloading all the data, he and Patrick Haynes (who's fully aware what he's up to) start communicating via sign language before he leaves to avoid their conversation being recorded. Does anyone know what they are saying to each other?

Gavin Jackson

Chosen answer: The scene is captioned as follows - Snowden: "I might not see you again." Haynes: "You 'gonna leave me here...with Captain America? Thanks a lot." Snowden: "NSA may come after you." Haynes: "I don't know what you're talking about. [walks away.]" I should note that my profession is that of an American Sign Language interpreter. From my observation, they are signing pretty much correctly, if not artfully. The only difference is that what the movie translates as "I don't know what you're talking about" would probably be more accurately stated as, "I don't understand your comment." And there really isn't a sign used for "gonna'." Hayes just says, "you're leaving me here with him? Captain America!"

Michael Albert

Thank you!

7th Aug 2017

The Terminator (1984)

Continuity mistake: When Sarah rolls Kyle's body over and looks at him, his face is covered in blood and his eyes are open. When Kyle is later zipped up in the body bag, his face is clean and his eyes are closed.

Gavin Jackson

4th Aug 2017

Baby Driver (2017)

Trivia: During the second heist, all of the crew are wearing Austin powers masks. Kevin Spacey who co-starred in this had a cameo appearance in the third Austin Powers film, playing himself.

Gavin Jackson

Question: If John Connor and the resistance are so good at reprogramming Terminators (They have done it 3 times during the series), why don't they just use them to help fight the war? Continuously sending them back in time may have kept Connor alive, but has achieved little else.

Gavin Jackson

Chosen answer: As long as Skynet kept sending other Terminators into the past to kill John Connor before he became the leader of the human resistance, it was necessary to keep sending reprogrammed Terminators into the past as well so that the timeline would not be altered, thus resulting in a Skynet victory. While it is possible that John and the resistance may have used reprogrammed Terminators to help them in the future, this was never depicted or otherwise mentioned within the film series.

zendaddy621

18th Jul 2017

Another 48 Hrs. (1990)

Factual error: No handgun shots can throw a police officer through the air halfway over a diner. That's just pure Hollywood fantasy (Mythbusters even proved this using all kinds of guns).

Gavin Jackson

18th Jul 2017

Payback (1999)

Other mistake: When Porter cuts the fuel line on the car and is about to light it, Carter's henchmen see him and act surprised. Despite having ample time to get out (as any rational person might do), they just sit in the car and wait for it to explode.

Gavin Jackson

Question: In the movie, Buford shoots two flying bottles whilst holding the gun at his hip. Is that even possible?

Gavin Jackson

Chosen answer: Technically, yes, but no matter how adept the shooter, such a trick would always rely more on luck than skill. Shooting from the hip makes it impossible to aim any better than "in the general direction of the target."

23rd Jun 2017

Xanadu (1980)

Chosen answer: It's implied that it could be her, but there's no definitive answer as to whether or not it is. It's deliberately left to the audience's interpretation.

raywest

12th Jun 2017

Runaway Train (1985)

Question: I am really confused by one scene. When Buford realises that Bandit is leaving the same service station he is at, he attempts to drive after him, but the front of his car is cranked up, When it cuts back to that scene, Junior (for no apparent reason) is lying on the hood and a crowd is watching them. Was there some missing scene which explains why Junior is in that odd position? Also why is Buford's car cranked up to begin with?

Gavin Jackson

Chosen answer: There's no missing scene; it's just a joke, albeit not particularly well-executed. Basically, the car was cranked up to replace the tires, and Buford forgot about them in his haste to pull out and crashed into the car in front of him. When we cut back, the joke is that he hit the other car so hard Junior flew out of his seat and wound up on the hood. The crowd gathers, as they tend to do in real life, around the accident to see what happened.

12th May 2017

Under Siege 2 (1995)

Stupidity: Dane is a brilliant man who has invested countless millions (hence his billion asking price) and much planning into the operation. Yet he only makes one copy of the disk? Suppose that disk had been lost or broken? His whole operation would have been a total waste.

Gavin Jackson

16th Feb 2017

Dragonheart (1996)

Other mistake: At the beginning when the king (Einon's father) grabs a torch and sets fire to the villagers' home, he only touches the corners of the roof. Yet seconds later, a fire suddenly begins inside the house...for no reason.

Gavin Jackson

Upvote valid corrections to help move entries into the corrections section.

Suggested correction: Someone inside panicked and knocked a candle over?

dizzyd

Question: When Bond is getting ready to go down the ski jump, he starts getting chewed out by the guy at the top. What is the guy saying to him? I've always assumed that he is demanding to know why Bond has only one ski pole...but that's just a vague guess.

Gavin Jackson

Answer: He's complaining that Bond didn't wait his turn for the ski run. He cut in front of everyone.

Chosen answer: I always thought that the guy in charge of the jump was saying that Bond didn't have the correct skis to do the jump or something. As I saw it Bond's skis were generic ones for cross country and / or regular skiers and to do the jump Bond needed, and didn't have, special, specific skis designed for jumping.

Alan Keddie

I'm more inclined to agree with you. It didn't appear like Bond had queue jumped. And I doubt the guy at the top would have known (or even cared). The guy did seem to be looking at Bond's attire, so I have to agree that he was chewing him out on that.

Gavin Jackson

12th Oct 2016

Superman III (1983)

Stupidity: When Superman gets to the bridge after the truck falls off, he asks the firemen if there's anything he can do. They responds that there's really nothing as he got there too late. How about asking him if he could get the truck out of the water. That would have been a help.

Gavin Jackson

8th Sep 2016

Star Wars (1977)

Question: When R2D2 is stunned by the Jawas and falls over, did Kenny baker do the stunt? Me and my brother have argued about this for years.

Gavin Jackson

Chosen answer: Most likely, not. The 'costume' Kenny Baker wore had what look like hoses running from the center body to the feet of R2 (to conceal his legs). Looking at the scene in question, those hoses cannot be seen. On that note, it looks like a third leg (and not the center one shown elsewhere) is visible behind R2 and probably used to push him over.

26th Jul 2016

Lethal Weapon 2 (1989)

Stupidity: When the guy is escaping on the helicopter at the beginning, the cops are all in position pointing their guns. Despite the baddie shooting at them, not one cop bothers to fire back (despite being fully within their right to). They just stand there and let him escape.

Gavin Jackson

Chosen answer: Vivo V3.

30th May 2016

The Pelican Brief (1993)

Plot hole: Darby and Gray are surprised to learn at the end that the guy who tried to kill her in New Orleans was Khamel. Given that Khamel was (a) a wanted terrorist, (b) the prime suspect in the judges' assassinations, (c) shot in a public area in front of many witnesses and (d) holding a recently used handgun when he died, this would have been breaking front page news along with his name on newspapers and news shows. Ridiculous to think they had to find out from Voyles who he was.

Gavin Jackson

8th Apr 2016

Licence to Kill (1989)

Chosen answer: It was to cause a diversion so Bond could transfer the money back onto the Wavecrest while everyone else was running about sorting out the mess from the crash.

Question: Why did Erik despise humans so much, even though it was a mutant who killed his mother?

Gavin Jackson

Chosen answer: Because it was a human who started the Holocaust and got Eric and his family put into a concentration camp in the first place. Seeing how one human reacted to just another race he saw what humans in power would do to another race or species the feared.

28th Feb 2016

Bad Influence (1990)

Question: After Alex has stolen his furniture and killed Claire, he says to Michael "OK, now we are even." So why did he continue to stalk him and plant evidence? What was his motivation there?

Gavin Jackson

Answer: Because that's what psychopaths do.

27th Feb 2016

Concussion (2015)

Answer: He was angry.

26th Jan 2016

Rapid Fire (1992)

Visible crew/equipment: When agent Stuart brings Jake before Serrano, we see Serrano sitting at a table. In every shot of him sitting, you can see the shadow of the camera on the wall at the right.

Gavin Jackson

14th Jan 2016

Goldeneye (1995)

Revealing mistake: At the beginning when Bond is chasing the plane on the motorbike, in the last shot of Bond from behind before he goes over the cliff, the background shot of the mountains is a painting. Not only can you see the square sections on the wall, but some parts of the painting don't even match. (00:09:25)

Gavin Jackson

Question: In both this film and The Desolation of Smaug. Legolas defies gravity in unbelievable and even comical ways. Is this a character trait, or just some ridiculous touch by the filmmakers?

Gavin Jackson

Chosen answer: Elves are described in the books and shown in the LotR trilogy doing things that would not necessarily rule this behavior out, ridiculous as it looks. For instance, when the Fellowship is trudging through the snow on Caradhras, Legolas is briefly shown walking easily on top of the snow while every one else's feet are sinking into it.

Phixius

18th Nov 2015

Analyze This (1999)

Question: When Nicky Shivers is being held hostage, De Niro is about to belt him in the face with a metal pipe and make him fall into a hole. This most likely would have been fatal. But De Niro has a change of heart and walks away. Then Nicky sardonically comments "Well geez, that was pretty painless." Given that he was just spared from a painful ending, why would he make such a smart-alec comment?

Gavin Jackson

Chosen answer: That's how he dealt with facing certain death and then not being killed. Some would be silently grateful as you suggest, others wouldn't. Everyone reacts differently.

Captain Defenestrator

18th Nov 2015

The Bodyguard (1992)

Question: I have two questions. Firstly after beating up the guy in the hotel kitchen, Frank returns to the room and tells Delaney that he is quitting. Why was he so ticked off? And secondly, if Nicki was the one who hired Portman to kill Rachel, why did Portman kill Nicki and why did he try to molest Rachel in the bedroom?

Gavin Jackson

Chosen answer: I think he was ticked off because he was being so personally affected by Rachel. He talks about being disciplined. He tries to keep emotions away from his job, but he liked Rachel. Also, Rachel at the time was angry with him, but also showed a lot of disregard for her own safety (as you remember, when he got back it was just after she and Tony went shopping without telling Frank). So yeah, I think it was frustration for the overall situation. To answer your second question, from the conversation with Frank at the lakeside house, Nikki had asked around at a club she went to, but she herself didn't hire Portman. Someone named Armando did the hiring.

Meg Charnley

2nd Nov 2015

Virtuosity (1995)

Chosen answer: 24.

19th Oct 2015

Die Hard 2 (1990)

Question: When McClane is crawling up into the elevator shaft and the news reporter mentions Colonel Stewart, McClane looks at her and says "Stewart, that's who it was." Did he mean "That's who it was on the speaker." Or "That's the guy I bumped into today"?

Gavin Jackson

Chosen answer: That's the guy he bumped into earlier, and didn't remember where he'd seen him from until she said his name.

Captain Defenestrator

1st Oct 2015

Jaws (1975)

Question: In one scene when the Orca is chasing after the shark so Quint can harpoon it a second time, Brody and Hooper are both smiling. What are they smiling for?

Gavin Jackson

Chosen answer: Adrenaline rush. Also, Hooper's thrilled to participate in the chase of the largest great white he's ever seen, while Brody's excited that they'll soon kill the shark that's claimed many of the lives he was supposed to protect.

17th Sep 2015

Under Siege 2 (1995)

Answer: I would say yes they where planning on killing some of the mercs. There is one fact that shows that. if they had plan on putting all the mercs into the copter, then the guys in the engine car should have know at x time get out and move back to get into the copter, giving the fact that they are still in the engine car when it hits the other train tells me they didn't know.

Chosen answer: Dane's speech was for the benefit of the hostages, not mercs. (It could be assumed that they knew he was lying.) Near the end of the film, a helicopter appears the train and prompts the mercs to board. The pilot's announcement of the first pickup (who was actually the porter) as well as the pilot's reaction to the fight between the porter and the female merc indicates that he expected to pick all of them up.

FleetCommand

Question: How is it even possible for Ultron to use Loki's sceptre to mind control the technician woman...considering that the sceptre came from Asgard and Loki is a god, whereas Ultron is a sentinel robot. Is this a goof?

Gavin Jackson

Chosen answer: Loki is only a god by our standards (commanding incomprehensible power, borderline immortal, etc.) The sceptre contains a mind stone that can presumably be controlled by any sentient being, and Ultron is most definitely that.

28th Jul 2015

Unforgiven (1992)

Question: Why was Little Bill so protective of the two cowboys that had bounties on their heads? And why was he so vengeful when they were both killed? He didn't know them.

Gavin Jackson

Chosen answer: I think Little Bill takes offense to this sort of thing taking place in his town on his watch. He tells Mr. Beauchamp while he is writing his biography something like, "I do not like assassins, or men of low character."

Answer: Little Bill wanted to be sure that the cowboys would live at least long enough to pay compensation to the owner of the saloon/brothel for disfiguring one of the prostitutes; Bill became furious when the cowboys were killed because they would never be able to make good on their agreement of compensation.

zendaddy621

The cowboys bring in their ponies and pay off their debt early in the film. "Davey Boy" even brings in an extra pony especially for Delilah, which he calls the "best in the lot." (Which the other whores promptly reject and begin throwing dung at them). Both cowboys are killed long after they pay their debt to Skinny.

There is a scene where the two cowboys come to town and pay the required compensation in horses.

Question: Given how dumb Biff is, how in the alternate 1985 when confronted by Marty did he remember his old self in 1955 warning him about Marty and Doc. Especially considering when he got the warning, he seemed to be paying very little attention and just walking off.

Gavin Jackson

Chosen answer: After young Biff won bet after bet after bet, he realized that the old Biff was obviously the real deal. As such, he undoubtedly racked his brain in an attempt to recall every detail he could about his encounter with his older self.

Phixius

Question: When Michael Palin is thrown into the fiery pit, he is either saying Yeeeoohhhhhhh or Yelllowwwww (meaning that was his favourite colour). Despite watching it many times, I'm still not sure which one it is...so does anyone know?

Gavin Jackson

Chosen answer: It's kind of both. He says "Blue", then realises he got it wrong and starts to say "No, yellow!" but then screams as he is thrown into the pit, so it's like "Blue. No, yell. Ooooooohhhhh!"

Sierra1

Answer: There was once a published Script. It was Yellow. The joke was he died because he changed his mind mid-answer.

9th Apr 2015

Heat (1995)

Question: I have two questions regarding the drive-in sequence where De Niro and his crew are double-crossed by Van Zant. Firstly what was the make and model of the white vehicle the deliver guy was driving? And secondly, who was the actor who played the driver?

Gavin Jackson

Answer: The driver was played by a guy called Vince Deadrick, Jr. (It's in the end credits crawl). All I know about the vehicle is it's a dodge.

Other mistake: When Bond is being chased on the bobsled track, just before he escapes the track.there is a shot of Eric Kriegler taking one last shot at him. In this shot however, Kriegler's bike appears to be in an open area and certainly not on a bobsled track. In fact it looks like a disused shot from when the chase started.

Gavin Jackson

13th Mar 2015

Halloween II (1981)

Question: Did Dick Warlock play Michael Myers in every scene he's in, even stunts? I was curious considering that Warlock was a stuntman and could have done the balcony fall and Michael burning scenes as well.

Gavin Jackson

Answer: From what I have read: yes.

Alan Keddie

3rd Mar 2015

Avatar (2009)

Question: At the end when all the forest animals on Pandora join in the fight, they seem to be targeting only humans. Since they can't determine the good ones from the bad ones, wouldn't this have put Norm Spellman (now back in his human form) in extreme danger? Always considered it a plot-hole, but any explanations would be welcome.

Gavin Jackson

Chosen answer: I wouldn't say it was a plot hole. Animals may be able to attack only humans, but they would be unable to differentiate between individual ones, so yes, Norm would be at risk of being attacked.

raywest

2nd Feb 2015

Runaway Train (1985)

Question: How was Sara so sure that there was no engineer on the train, considering that she was not able to get to the front carriage to find out?

Gavin Jackson

Chosen answer: Most likely from the speed at which the train was going, a train is like driving any large vehicle. You have to maintain a certain pace, to stop at an instant, for the unexpected.

24th Jan 2015

Gleaming the Cube (1989)

Question: In the later scene where the two Asian gang members are being interrogated to discover who hired them, they jokingly say something to each other in their own language, the translator cop makes some smug comment back, and then reveals to Steven Bauer that their employer was Lauderdale. Is anyone able to translate what was said during this little exchange?

Gavin Jackson

Answer: More than likely, it was probably something made up and meaningless. But of course, we would never know because it's Vietnamese. The best you can do is have a Vietnamese person view that part of the film and get them to translate it if it was actually a real sentence being spoken, which I doubt.

Question: Why didn't Charles make any attempt to rescue Wolverine at the end? With Cerebro's help he wouldn't have had any trouble finding him.

Gavin Jackson

Chosen answer: Who's to say he didn't later attempt to find him? Wolverine was flung quite a distance away from where Charles was and he didn't have Cerebro with him to immediately locate him.

Phaneron

Chosen answer: According to his wife, the painting was made specifically for the movie. Robert doesn't paint at all.

Answer: He's a surfer dude, he wanted to be polishing his boards for that scene.

Question: Would Phoenix have been capable of destroying all the sentinels in the future timeline?

Gavin Jackson

Chosen answer: Yes. But she was already dead in this film.

MasterOfAll

1st Nov 2014

Rango (2011)

Chosen answer: Because Hawks are predators and rattlesnakes are not uncommonly their prey. It's more of a reference to real-life than based in any in-movie logic.

Phixius

Answer: It also was the fact that Jake had a fear of hawks. When he spotted one fake or real he would just hide without thinking. Metal beak also didn't think like the other animals. She had a primal mindset which would scare all the other animals.

8th Oct 2014

The Flintstones (1994)

Question: On Fred's first day of his new promotion, he says "I'm only one man, Barney." To which Barney responds."Not from the back" and they both laugh. Could someone please explain this gag to me. I have tried for years to understand it, with no luck.

Gavin Jackson

Chosen answer: He means Fred is fat. That from behind, he looks big enough to be two or more people.

Quantom X

17th Sep 2014

Near Dark (1987)

Plot hole: In the morning at the motel, the vampires have a big shoot-out with local law enforcement, which results in one at least one cop being killed. But when they escape, the police don't bother pursuing them or even giving them a another thought. Even for an 80's film, this is beyond ludicrous.

Gavin Jackson

Question: What could possibly have made Teasle think that it was Rambo who killed Gault? Teasle was fully aware that Rambo was injured and unarmed and that Gault was in a helicopter trying desperately to shoot him (against orders). So how the hell did he come to that conclusion?

Gavin Jackson

Chosen answer: Rambo essentially admitted responsibility, when he tried to make peace with the officers as he stood near the body. As well, we can assume the chopper pilot reported the incident - while he may not have seen Rambo, rocks don't throw themselves at helicopters.

DavidRTurner

Answer: In Teasle's mind, anything that happened after Rambo escaped would be his fault - he put in motion the sequence of events that lead to everything happening. Additionally, he isn't exactly in his right mind with all this.

Question: Why did Elliot let all the frogs out? And why was he asking one if it could talk? I realise he was drunk (or ET was), but this doesn't explain such behaviour.

Gavin Jackson

Chosen answer: Elliot, through a psychic connection to ET, is experiencing the effects of the beer that ET is drinking. It has impaired Elliott's judgment and unleashes his inhibitions. He frees the frogs because he doesn't want them to be killed and dissected in class. Talking to one is just a side-effect of being intoxicated and he may be identifying it as a sentient being like E.T.

raywest

Question: Could someone please tell me where all the other walking apes/chimpanzees/orangutans came from? Caesar was the only surviving ape from the last film, so where did all the rest come from?

Gavin Jackson

Chosen answer: Caesar wasn't the only surviving ape, just the only surviving talking ape. Conquest of the Planet of the Apes is set roughly 20 years after Escape from the Planet of the Apes, and in that time man began to keep apes as pets (disease wiped out dogs and cats) and (most likely through breeding), the apes evolved into what we see in the film.

Bishop73

8th Aug 2014

Young Guns (1988)

Question: Did that bounty hunter in the bar ever realise that he was in fact talking to Billy the Kid... Prior to being shot?

Gavin Jackson

Chosen answer: He may have. When Billy started whistling, the bounty hunter's face changes. He continues to speak to Billy in a dismissive tone, but does eventually pull the trigger (although Billy had unloaded the pistol). Hard to believe he would just shoot some mouthy kid in a bar.

Question: I have two questions. Firstly I read that Liev Schrieber makes an appearance. Does anyone know which one he is? Secondly, given how much Hunt admired Jake, why didn't he warn him that the gang had a new leader and that he risked death by going back due to his betrayal? Seems odd to me that he said nothing.

Gavin Jackson

Answer: Don't recall seeing Liev, as for your second question, maybe Hunt figured it was nothing Jake didn't already know, or at least conclude.

6th Jul 2014

Blazing Saddles (1974)

Question: At the beginning, Lyle refers to the song Camptown races as "The Camptown lady"? Is this simply cause he's stupid, or is there any other reason?

Gavin Jackson

Chosen answer: The opening line of the song refers to the Camptown Ladies and the phrase "Camptown Races" never appears anywhere in the lyrics. If nobody told him otherwise, Lyle may simply have assumed that some variation on "Camptown Ladies" was the actual title.

Tailkinker

The actual title of the song was "Gwine to Run All Night, or De Camptown Races," written by American lyricist Stephen Foster and first published in 1850. Over many years on the minstrel show circuit, the title was shortened to "Camptown Races" and was sometimes erroneously called "Camptown Ladies." While the phrase "Camptown Races" doesn't appear in the lyrics, the phrase "Camptown Racetrack" does appear in the second line: "Camptown ladies sing dis song, doo-dah, doo-dah, Camptown Racetrack five miles long, oh-de-doo-dah-day." The song refers to Camptown, Pennsylvania, a real town with a popular horserace in the mid-1800s.

Charles Austin Miller

26th Jun 2014

The Woman in Red (1984)

Question: How did Miss Milner know that it was Teddy who (unintentionally) asked her out on a date? She couldn't see him and he didn't introduce himself so I am very confused.

Gavin Jackson

Answer: She had a secret lust for him. She was the office (Sex and the City) Samantha Jones. Plus they were married in real life, so the joke was that he despised her, while she wanted him.

24th Jun 2014

Ghostbusters (1984)

Chosen answer: For scenes like when Bill Murray is covered with it and it gets in his mouth, it's gelatin mixed with food coloring. For other scenes it's various plastics.

9th Jun 2014

General questions

I saw a film not long ago but can't remember the title. It features a bumbling spy, and in one scene when he is trying to break into a place, he pulls out a credit card, types some numbers on the side and it emits a laser beam. When he is finished, he can't turn the laser off properly and it burns a hole in his pants. Does anyone know this film?

Gavin Jackson

Chosen answer: I haven't seen the film in a few years, but I believe it's from the movie Johnny English. Starring Rowan Atkinson.

Question: Could somebody tell me once and for all why every mutant in these films have nicknames?

Gavin Jackson

Chosen answer: Because just like in the comic books, all super heroes and villains have names that are a representation of either the abilities they possess, paraphernalia they use, their personalities, and/or the circumstances of their origin. Magneto is a much more compelling and memorable name for a character than simply calling him Erik. Presumably it also avoids confusion if there are several mutants with the same "regular" name, to have a unique nickname for each one.

Phaneron

Answer: In addition to what was said, the nicknames also provide the individual mutant a way to identity more closely with the fact that they are mutants. As Mystique said in "X-Men: The Last Stand" when addressed as Raven, "I don't answer to my slave name." Many mutants are rejected by their parents when they find out they're mutants, so the "nickname" serves as a way to distance themselves from those that rejected them. In addition, at times their nicknames are called "code names." In this case, it would be a way to address a mutant without using their real identity, in essence a secret identity that's common in almost all superheroes and super villains.

Bishop73

Question: Joan wilder is a successful author with a nice apartment and is financially well off. So why wouldn't she pay Jack the $500 he originally wanted for his help? Given her situation, I didn't think it was too much to ask.

Gavin Jackson

Chosen answer: She's lost in Columbia and doesn't have access to money beyond what she has with her. She doesn't want to give him everything she has because she doesn't know when she'll be able to get more.

14th May 2014

Speed (1994)

Question: What did Harry mean when he said to Jack "And I got shot... Another few inches and they would have given the medal to my wife." I still can't make complete sense of that line.

Gavin Jackson

Chosen answer: Jack shot Harry in the upper leg, and Harry is presumably referring to the proximity of the femoral artery, a large and vital blood vessel in the thigh. If Jack's shot had hit him there, he would almost certainly have died within minutes, and they would have to give the bravery medal to his widow.

Sierra1

23rd Apr 2014

The Client (1994)

Factual error: When Mark pranks the detective by using his credit card to order 20 pizzas, he only reads out the card number on the phone. To order via credit card, you must say the expiry date as well.

Gavin Jackson

22nd Mar 2014

Lone Wolf McQuade (1983)

Question: Why didn't Kayo get any credit at the end for his help? Mcquade and Jackson got a rousing public celebration for their work...but Kayo got nothing. I never understood that.

Gavin Jackson

Chosen answer: Part of it could be because more people in the crowd know McQuade or Jackson personally. We know McQuade's daughter and ex-wife are there. However, Kayo is also given the Texas Award of Valor, same as McQuade and Jackson.

Bishop73

18th Mar 2014

Death Race 2000 (1975)

Question: I have two questions. Firstly, was getting to shake the presidents hand all the winners got, or was there anything else? And secondly, were the drivers allowed to kill the other competitors as well? Just seems odd to me that the officials who kept track of every killing in the race covered up the fact that Frankenstein had killed Joe Viterbo at the end (blaming the resistance instead).

Gavin Jackson

Chosen answer: Like the Olympic gold medal winners, the winner gets t.v. exposure, acting roles, product endorsements, VIP treatment into exclusive parties and bragging rights. By blaming the Resistance, that paints them as the villains and keeps the race going.

24th Feb 2014

Stir Crazy (1980)

Chosen answer: Because his employers discovered he had put grass (marijuana) in the salad and got all the dinner guests high. Well, he didn't, the cook did. But he brought it into the house so he was fired.

24th Feb 2014

RoboCop (2014)

Chosen answer: He most likely has an artificial heart or some type of system that circulates his blood, this system doesn't necessarily have to take the shape of a typical human heart. However if his biological heart is still functional, it may be behind the lungs.

Answer: Watch again the scene (s), especially close to the end when his armor is reverted to silver and just between his lungs; his heart is just behind his lungs and beating, though if you blink you might miss it.

Erik M.

Except the heart is located in front of the lungs, not behind them.

lionhead

Why Murphy's heart isn't where it should be in a normal human is up to speculation-perhaps the transformation into a cyborg necessitated it's being repositioned, perhaps. The heart is there, though.

Erik M.

Question: When Hauk reads Plissken's military history to him, he refers to him as SD Plissken. Does anyone know what the D stands for?

Gavin Jackson

Chosen answer: I searched this character on the internet, and it's mentioned in several places that Plisskin's full name is never given.

raywest

9th Nov 2013

Flash Gordon (1980)

Question: Why did Prince Barron hate Flash so much? His burning desire to kill this complete stranger left me somewhat baffled.

Gavin Jackson

Chosen answer: Prince Barin was the fiance of Princess Aura, who took a shine to Flash. He was quite simply jealous.

Ioreth

23rd Oct 2013

The Last Stand (2013)

Revealing mistake: When Gabriel Cortez reaches the bridge at the end, he sees Arnold standing there waiting for him. It's blatantly obvious in this shot that all he is looking at is a rear projection backdrop of Arnold and the background. (01:25:40)

Gavin Jackson

Chosen answer: He's initially playing Missile Command, then he goes to play After Burner.

envisaged0ne

2nd Sep 2013

Airplane (1980)

Question: In the film Elaine is going around taking dinner orders on paper like a waitress which has never been the way airlines do it. They bring the cart down the aisle and then ask what you want. So my question is, is this an in-joke, a goof, or did some airlines really do it this way back then?

Gavin Jackson

Chosen answer: Airlines have done this in the past. Remember that back in the late 70's when this was filmed, flying was still somewhat of a rare occasion for a lot of people, and airlines treated customers much better.

LorgSkyegon

29th Aug 2013

Fast & Furious 6 (2013)

Chosen answer: All the flashbacks are archive footage.

Ssiscool

Question: One of the taglines for this film is "It's nothing personal". I have no idea what that has to do with the film and was hoping someone could explain it.

Gavin Jackson

Chosen answer: Two possibilities. 1: The Terminator is emotionless, so the killing isn't personal, but rather what it's programmed to do. 2: Sarah Connor's plan to kill Miles Dyson to stop Skynet's creation.

Captain Defenestrator

It's also a sly nod to another famous tagline, Jaws: The Revenge. "This time it's personal."

BaconIsMyBFF

Chosen answer: Their score was pretty bad, but just good enough to win the bet. Getting the money was the important part - they didn't really expect to win the competition.

3rd Jul 2013

Superman (1978)

Question: Jonathan Kent dies of a heart attack. So why does he look at his arm in shock before he collapses? What does that tell him?

Gavin Jackson

Chosen answer: Massive heart attacks are often preceded by pain in the left arm before it is felt elsewhere. The aorta, the main artery leading away from the heart runs down the left arm. There can also be pain in the right arm, but the left one is more common.

raywest

5th Jun 2013

Stripes (1981)

Question: Given that Sergeant Hulka was only the platoon's drill sergeant, that means his authority should have been done once they graduated from basic training. So why did he turn up in Germany and continue to give them orders?

Gavin Jackson

Chosen answer: According to the movie's premise and plot, once they (the platoon) impressed the brass at graduation and was assigned to the Germany gig with the "EM-50", Sgt. Hulka would naturally be with them as he was their original commanding officer. Plus, it would be logical to suggest that he would personally assign himself the detail of overseeing his platoon as they were his to begin with.

CCARNI

He's assigned as a Drill Sergeant, that is his job. Once he was fully healed he would have been assigned another group of recruits.

Answer: He wanted to make sure that they don't screw up this very important assignment, which they did.

Question: OK I have a few questions about this. (1) Who blew up the car with Hannah? Was it Jim or Clare? (2) Ethan taunts Krieger because he has the real NOC list, but was it also because Ethan knew he was a traitor and wanted him to walk out? (3) At the end when Ethan is disguised as Phelps, he is wearing the same trenchcoat as the real Phelps (who then shows up). Is this because he knew Jim was onboard, or simply cause he knew how his old boss loved to dress? (4) And lastly after Clare is killed, Hunt acts devastated and goes after Jim in anger. Given that she was a traitor who was always on Jim's side (and might even have killed Hannah), why would Hunt feel this way at all?

Gavin Jackson

Chosen answer: 1) Jim most likely did it when he emerged from the water (as seen in the flashbacks/Ethan's theory, but it's hard to be 100% certain). 2) Ethan was just fooling Krieger. He didn't know he was a traitor yet. 3) Ethan likely knew exactly what Jim was wearing, as he planned the whole thing all along. 4) He's upset because Jim just murdered his wife to escape, someone who until very recently Ethan still considered his friend. You can't just turn loyalty off like that.

LorgSkyegon

Answer: Claire killed Hannah. Ethan didn't want to believe it. Jim had no way of knowing when Hannah was in the car.

15th May 2013

Vegas Vacation (1997)

Chosen answer: No it isn't. Brinkley has three kids. One born in 1985, one born in 1995 and one born in 1998. Vegas Vacation was made in 1997. This was before her third child but 2 years after her second child. The kid in the car is shown as a baby, not a two year old.

oddy knocky

21st Apr 2013

Jurassic Park (1993)

Question: Why would Alan test an electric fence by grabbing it with both hands? That's like clicking a gun at your head to see if it's loaded. Couldn't he have tried kicking it or maybe tapping it with his hand?

Gavin Jackson

Chosen answer: Alan already knew the electricity wasn't turned on when he grabbed it. He first tested the fence by throwing a stick at it and nothing happened. He could also see that the warning lights on the fence post were off. He only touched the fence cables to play a joke on the two kids, to lighten the mood a little.

raywest

Throwing the stick was meaningless and would not have told him a thing. Perhaps, he did so to set the kids up for his questionable joke, given what they'd been through and still were in the middle of.

Answer: The stick testing the fence would not work because wood does not conduct electricity.

Wood contains a good deal of moisture unless it has been completely dried out. That's why electric transmission companies and local electric distribution companies cut branches away from power lines and transmission lines. That said. The stick would have to touch a wire and ground simultaneously. In the movie it was thrown against the fence but was not grounded. I don't think I would have used this test.

17th Apr 2013

Jurassic Park (1993)

Continuity mistake: When the car with Timmy in it falls down the wall, it lands in a tree flat again the concrete wall. Yet later when Alan is rescuing Tim, this is no wall to be seen anywhere. Only other nearby trees.

Gavin Jackson

25th Mar 2013

The Night Flier (1997)

Question: Probably a dumb question, but is this the only film in which Miguel Ferrer has ever received top billing? Usually in everything I have seen him in, he is in a supporting role, so I was just wondering.

Gavin Jackson

Chosen answer: He's received top billing at least 6 times, most notably "Hard Ride to Hell" and "The Harvest", but he's also shared in top billing another half dozen times or so.

Bishop73

11th Mar 2013

Police Academy (1984)

Question: Why did Mahoney get a medal at the end? All he did was run up a building and got himself taken hostage. Hightower deserved his cause he knocked the guy out, saved Harris and Mahoney and defused the situation. Even if it was for bravery or heroism, Mahoney achieved nothing with his actions and shouldn't have been rewarded.

Gavin Jackson

Chosen answer: Just because you fail in your attempt at heroics doesn't mean you weren't brave as hell for trying. Mahoney knowingly put himself in a very dangerous situation with the sole intent of saving two of his fellow officers. That's bravery.

LorgSkyegon

3rd Mar 2013

The Doors (1991)

Question: Did Jim Morrison really go around falsely telling reporters that both his parents were dead? Or was that just an Oliver Stone embellishment?

Gavin Jackson

Chosen answer: That is actually true. Morrison grew up in a military family and his stern parents harshly disciplined their two sons by verbally assaulting them until they broke down in tears, were utterly humiliated and admitted to wrong-doing. Morrison broke off almost all contact with his family after graduating from college.

raywest

18th Feb 2013

The Terminator (1984)

Question: Some people think Ginger is Sarah's sister, but I have always thought she was just her close friend and roommate. Who's right?

Gavin Jackson

Answer: Ginger is Sarah's roommate and best friend.

Chosen answer: No mention is made of her being Sarah's sister. If she was, at some point in the other films, it would have been mentioned.

Captain Defenestrator

Question: I have two questions about this film. Firstly, was the black woman Hushpuppy was dancing with at the strip bar her biological mum? And secondly, were the huge pigs (whatever they are called) that appear at the end really there. Or were they simply imagined by Hushpuppy? Because the other kids responded to them, but they seemed way too oversized to be real.

Gavin Jackson

Chosen answer: Both questions are left open-ended. It is unlikely, but not impossible, for Hushpuppy to have found her mother (although I believe the music playing at that moment is a hint that she is a look-alike). The beasts are supernatural creatures; up to you whether they are literal or in her mind.

Question: When Bond goes to the fighting ring and discovers that Maud Adams is dead, was that really a dummy of her? It sure looked like one to me.

Gavin Jackson

Chosen answer: No, it was the actor.

28th Dec 2012

Blue Thunder (1983)

Question: Why didn't Murphy tell his boss about Cochrane's plan to kill him? He tries to when he gets back, but notices some strange behaviour and walks off. Is it because he (a) doesn't trust the department and thinks the boss is in on it (b) doesn't think he will believe him (since Lymangood hid the proof) (c) decides to deal with it himself (d) just the writers' way of advancing the plot.

Gavin Jackson

Answer: Yes to A, B, and C.

10th Dec 2012

Star Wars (1977)

Question: Did that officer who Vader attempts to choke, throw up on the table afterwards? I can't tell if that's vomit or just some buttons on the table, but as I haven't watched the film in some time, any help would be great.

Gavin Jackson

Chosen answer: Every person on the table has a monitor in front of them, and what you see is a yellow button or some kind of mouse or joystick that glows.

Anastasios Anastasatos

10th Dec 2012

Toy Story 3 (2010)

Question: In Cars 2, it was decided not to bring back Doc Hudsen out of respect for the late Paul Newman. So why was slinky recast for this film? Didn't the filmmakers feel that Jim Varney deserved the same respect? Considering they didn't bring back certain characters like Bo-peep etc, I find it insulting that they felt so little of Varney.

Gavin Jackson

Chosen answer: Well, with all due respect to Mr. Varney, he's no Paul Newman. You respect different actors in different ways. Paul Newman is much more of a "remember in silence" type actor, when Varney's much more relaxed and silly style lends itself more toward celebrating the character over the actor, in which case removing the character would not necessarily be the best decision. I don't know if these were things the Pixar crew specifically thought of, but that's how I view it.

Garlonuss

22nd Nov 2012

General questions

I was wondering if anyone could back me up on this. When the James Bond films were in limbo between 1989 and 1995 and two studios were fighting over rights issues, I am sure I read articles which claimed that producer Joel silver was desperately trying to obtain the franchise with the full intent of casting Mel Gibson as Bond (despite Gibson repeatedly saying that he wasn't interested). I have found nothing anywhere online to back this up so I was wondering if anyone else had read this. Or it is simply my imagination?

Gavin Jackson

Chosen answer: There were many actors who were considered for the role of James Bond during this period, including Mel Gibson in 1987. MGM suggested casting him, but producer Albert Broccoli rejected him because he was not British.

raywest

Question: I was hoping someone could clear this up for me. Did ET really die and somehow return, or did he simply fake his death to fool the authorities?

Gavin Jackson

Chosen answer: First, it is hard to know whether he actually died and was revived, or possibly got so close to dead that the scientists simply assumed he had died and called it. But it was not a ploy. His link to the flower reflects his dying condition. As he died, so did the flower.

Garlonuss

8th Nov 2012

Psycho II (1983)

Other mistake: When Norman and Mary are at the swamp, the sheriff hands him the ashtray they found and bates explains that it belonged to Mr Toomey. Mary say "Who"? and the sheriff explains who he is. This makes no sense as Mary knew full well who Toomey was and had no reason to play dumb.

Gavin Jackson

Upvote valid corrections to help move entries into the corrections section.

Suggested correction: She wasn't playing dumb. She says WHO because she didn't hear the name correctly.

Question: This has been a question that's always brought up. Was Ellie Grimbridge always a robot from the very beginning, or was she turned into (or exchanged with) one when she was held captive at the shamrock factory?

Gavin Jackson

Chosen answer: She was turned into one when she was captured. The robots didn't speak and if you notice that after he finds her, she never says a word. As well, she displayed genuine emotion throughout the film before she was captured, something the robots also never did.

I agree, she was a real person until she was captured. If she were a robot, I think Chalice would have found that out when they made love. Plus, as you mention, she had real emotions until her robot replacement was rescued.

Answer: Another theory might be that Ellie has always been a robot but an advanced model that Cochrane designed to interact with the outside world.

19th Sep 2012

The A-Team (1983)

13th Aug 2012

Cool Runnings (1993)

Question: Could someone as overweight and unhealthy as John Candy's character really have lived in Jamaica? I would have thought the extreme hot weather conditions would have been rather dangerous for someone in his shape.

Gavin Jackson

Chosen answer: It may not be the wisest place to live, but that doesn't mean that he's physiologically incapable of living there, even relatively long term. It's pretty dangerous being that size anywhere in the world, as Candy's sadly premature demise illustrates; living somewhere as warm as the Caribbean probably wouldn't make the situation significantly worse than it already is.

Tailkinker

Chosen answer: Most likely a coincidence. It's a common Middle Eastern name.

raywest

8th Aug 2012

Robocop (1987)

Question: When the Swat team leader is trying negotiate with the guy in the Mayor's office, he says to him "Hey no problem Miller. Let the mayor go. We'll even throw in a Blaupunkt". Could someone please explain what a Blaupunkt is? I've never heard that word and its not in my dictionary.

Gavin Jackson

Chosen answer: Blaupunkt is a major manufacturer of car stereos. Miller wants a car as part of his ransom demands, so the SWAT team leader is playing along with those demands telling him that they'll make sure the car he wants is fitted with a really cool stereo.

Tailkinker

8th Aug 2012

Con Air (1997)

Question: Why did Poe get back onto the plane when it left Carson city? I know he stayed on after the first stop cause Cyrus wouldn't let Baby-o off and Poe needed to get him his insulin shot. But he got his shot in Carson city and Poe had no reason to get back on (seemed even more ludicrous after he tied the plane up.but anyway).

Gavin Jackson

Chosen answer: Poe is an ex-Ranger, and he wants to stop the criminals. Also he wanted to save the cops inside the airplane.

Anastasios Anastasatos

Chosen answer: A meal in the 80s would have cost 4 or 5 dollars. A bus ticket would have been around 50, or more, depending on where he was going.

Captain Defenestrator

23rd Jul 2012

The Client (1994)

Question: When Reggie is negotiating with Reverend Roy and tells him that Marks mum wants a walk-in closet, Roy responds with "Walk-in. Is that hyphenated?" Although I gather that Roy was being sarcastic, I still have no idea what that comment means. Any help would be great.

Gavin Jackson

Chosen answer: Meaning is the word walk-in hyphenated.... walkin or walk-in.

shortdanzr

Answer: I saw his question "Is walk-in hyphenated?" as a touch of humor.

21st May 2012

Titanic (1997)

Question: I have two questions about this film. Firstly who was the guy who tries to drown Rose after the Titanic sinks? I couldn't see his face properly and wondered if anyone knew who it was. Secondly, given how possessive Cal was of Rose, why didn't it seem to bother him when Jack leads her arm in arm into the dining room. It's almost like he just doesn't care in that scene.

Gavin Jackson

Chosen answer: The guy who nearly drowned Rose was just a random panicked passenger who needed something to hold on to, like a life ring. As for dinner, at this time, it was actually customary in higher society for a man to escort someone other than his own wife or fiance to the table at a dinner party, and always arm-in-arm. Husbands and wives (and fiances) were also not seated next to each other at the dining table, primarily to mix up the group dynamics and stimulate conversation. Being as Jack is the invited guest for having saved Rose, it would be acceptable that he should have the honor of escorting her into the dining room. At this point, Cal would hardly consider Jack a serious threat to his relationship with Rose, and would have no reason to object.

raywest

4th May 2012

The Avengers (2012)

Chosen answer: As she was extremely pregnant at the time of shooting, which would have been impossible to hide, it's likely that her participation was never considered seriously. Given the desire to get the film out on their chosen release date, there would have been no question of delaying shooting to include her in what could only have been a minor supporting role at most.

Tailkinker

25th Apr 2012

The Howling (1981)

Question: Why did Eddie Quist give Chris his rifle back before transforming? Is it because he wanted to die, or because he (wrongfully) thought the gun would have no effect on him?

Gavin Jackson

Chosen answer: The reason Eddie gave Chris back his rifle before transforming was probably because he thought the gun was loaded with normal bullets. Regular bullets (at least in this movie) do not kill werewolves. This is shown in an earlier scene with Eddie and Karen White when he confronts her about the police raid in the porno theater. The police shoot him down and it was thought that Eddie was dead. However, later it is shown that Eddie had escaped from the morgue. Also, right before transforming, Eddie digs the bullet out of his head. It was likely that Eddie was taunting Chris by daring him to shoot him, thinking that the gun would have no effect on him. What Eddie didn't know was that Chris had loaded the rifle with silver bullets, which are deadly to werewolves. If you look closely, you can see the shock on Eddie's face when Chris shoots him in the throat.

Chosen answer: Rusty was just being an ass. That's his character, him and his dad are usually going at one another. They really play up this relationship in European vacation.

Carl Missouri

Answer: Maybe, too, because he inadvertently killed her dog so maybe that killed her, too.

Kitty1019

4th Apr 2012

The Goodies (1970)

Kitten Kong - S2-E7

Trivia: On many recent specials and Live-shows, Tim has mentioned that in the scene where the Cat is dragging him face-down along the ground in the park a dog came out of nowhere and laid some do-do in his path. Since Tim didn't want to ruin the scene, he decided to continuously move himself right over it. Clearly it's something he hasn't forgotten.

Gavin Jackson

Chosen answer: It was said in the movie commentary that the drawing was the work of digital editing.

25th Mar 2012

The Hunger Games (2012)

Question: When Katniss is attacked by Clove after she takes the bag, why didn't she grab the knife she had (the one she cut the tracker-jacker nest's branch with) and use that instead of trying to fight her bare-handed?

Gavin Jackson

Chosen answer: In the book, she left it with Peeta in case he was discovered while she was away. Apparently she was meant to have done this in the film as well, they just neglected to show it, causing some confusion.

Phixius

Answer: She shows little to no competence with a knife. Katniss' ideal weapon is a bow, and she would not learn much on how to use a knife in the training time, especially as we have seen her do various other things in training. Though she could be able to cut off a still branch that in and of itself can't fight back, here she is dealing with a moving, killing opponent. She just might not know how to fight with a knife, or not be confident enough to do so.

18th Mar 2012

Godzilla (1998)

Continuity mistake: When Godzilla goes to eat the pile of fish, it only takes about 3 mouthfuls from the top of the pile before being chased off. In the later shot, all the fish have been eaten.

Gavin Jackson

13th Mar 2012

Licence to Kill (1989)

Question: After Sanchez has killed Krest, he goes to Bond and pays him for his information. Bond then says to him something like "only one man? No one would go after you alone." Did this mean that Bond was trying to get Sanchez to kill all his own men? Seems a bit ambitious to me.

Gavin Jackson

Chosen answer: It's more likely that Bond is trying to instill fear and paranoia into Sanchez, getting him to suspect that everyone is trying to kill him so that he trusts no one.

raywest

Continuity mistake: At the rebel base, Leia is informed that there is nothing more can do about Han and Luke until the morning and agrees to the hangar doors being closed. In the wide shot, you can see the doors starting to close, although the code hasn't been put in yet. In the next shot the code is put in and the doors start closing again. (00:12:10)

Gavin Jackson

12th Feb 2012

The Last Chase (1981)

Factual error: Franklyn Hart keeps his car running by pumping out the gas from the bottom of abandoned gas pumps which the pumps themselves couldn't reach. In reality, as this gas is on the bottom, it would be full of sludge and garbage, and since it has been sitting there for 20 years, it would barely be usable and would probably damage the engine. He certainly wouldn't be able to do high speeds with it.

Gavin Jackson

2nd Feb 2012

Pulp Fiction (1994)

Question: Are there any actual restaurants like Jack Rabbit Slim's out there, where booths are done up like cars? I know the restaurant they filmed in was torn down afterwards and I can't find anything online, so any help would be appreciated.

Gavin Jackson

Chosen answer: Disney World has one.

Grumpy Scot

31st Jan 2012

The Last Chase (1981)

Visible crew/equipment: After Capt Williams destroys the buildings, Hart briefly stops. As he drives off, the reflection of the camera can be seen on the ground nearby. VHS version only.

Gavin Jackson

31st Jan 2012

The Last Chase (1981)

Other mistake: Towards the end, Capt Williams blows up some buildings (or caravans) to try and stop Hart from continuing, and flies off. When Hart later stops, the same buildings are blowing up again. One could argue that it's being seen from a different perspective, but in the wide shot after Williams flies off, the car cannot be seen.

Gavin Jackson

23rd Jan 2012

The Last Chase (1981)

Visible crew/equipment: Near the end when Capt William sets his plane down behind the car and swoops over them, in one of the shots of him landing, you can briefly see the reflection of the plane filming the scene nearby. VHS version only.

Gavin Jackson

18th Nov 2011

Batman (1989)

Question: Probably a dumb question, but when I first saw this film.I thought that the opening scene where a young family is mugged in an alleyway was supposed to be a flashback to when a young Bruce Wayne lost his parents. Of course we know it isn't, but is that what the filmmakers wanted us to think. Or was it just my way of thinking?

Gavin Jackson

Chosen answer: It foreshadows what will be learned about what happened to Bruce Wayne's family and what set Batman on his path.

raywest

Answer: It's not just you. No matter how many times I watch I find it refreshing that this Batman movie doesn't start off by making us watch Bruce's parents get murdered for the dozenth time. But fans of the character definitely would have fallen for it in the theater, so it's like a clever inside joke for their benefit, I think.

CountArtha

18th Nov 2011

1408 (2007)

Question: Do continuity mistakes really count for this film considering it is a haunted room and one could say that the room itself is moving things around or manipulating objects?

Gavin Jackson

Chosen answer: There would have to be a point made within the movie of things being different from shot to shot. Otherwise it would have to be considered a mistake.

Phixius

Question: Has there ever been a backstory written for Jaws? I would love to know where he came from, and how he came to be, so I was wondering if there has ever been one written, and where I can find it.

Gavin Jackson

Chosen answer: Yes, there was a backstory for the character of Jaws in Christopher Wood's novelisation of the film "James Bond, The Spy Who Loved Me", not to be confused with the Ian Fleming novel.

Sierra1

Other mistake: When the helicopter is chasing the Lotus Esprit, it is flying fast, doing sharp turns and firing bullets. Even for the most experienced pilots, this would take a lot of concentration and hard work and would result in some stress. But when Bond spots Naomi in the pilot's seat, she is totally relaxed, stress free and barely looks like she's doing anything.

Gavin Jackson

Upvote valid corrections to help move entries into the corrections section.

Suggested correction: Not a mistake; this is movie-land, after all. When Bond spots Naomi, and they share an awkward smile, both are travelling in a straight line at that moment. No reason why she couldn't spare a few seconds to taunt her target.

DEvans

Indeed. Also, Naomi's working for a megalomaniacal madman as a secretary and an assassin. It's perfectly reasonable she's a stone-cold psychopath.

Jukka Nurmi

11th Nov 2011

Deep Blue Sea (1999)

Factual error: There's no way those dynamite sticks could have produced anything like the humungous explosion at the end of the film. In fact, the most they would have done would have been putting a hole in the shark's skin. (Proven on Mythbusters.)

Gavin Jackson

Question: How does the Liparus explode and sink at the end? After the nuclear subs have been destroyed, the ship just starts blowing up for no apparent reason. Was there any explanation for it?

Gavin Jackson

Chosen answer: When the nuclear subs have been destroyed, there's an explosion in the control room, and one of the men cries out "Fuel tank!" Liparus has numerous internal fires raging after the battle with her crew. These uncontrolled fires eventually spread to her fuel tanks and ammo storages, one of which explodes in a huge fireball and finally causes Liparus to sink.

I thought the final explosions that sank the Liparus were deliberate self destruction after completing the mission in an attempt to destroy any evidence, rather like when Bolfeld manually triggers explosives after being thwarted in You Only Live Twice.

Liparus sinks slowly, and that would leave her crew plenty of time to evacuate. Angry and armed crew with a grudge against Stromberg.

Jukka Nurmi

Plot hole: After sneaking into Zorin's secret laboratory, why did Bond and Tibbett decide to spend another day there? They would have known that the two guards they fought would inform Zorin they were spies and blow their cover. Besides, they already knew how Zorin was winning the races, so staying another day was just suicide. (00:39:40)

Gavin Jackson

Factual error: It would have been impossible for Zorin's airship to be inflated at the speed the film depicts. Nothing could inflate it that fast. (01:49:30)

Gavin Jackson

Continuity mistake: When we see Mayday jump from the Eiffel Tower in close-up, it's only a rather tiny jump (and would probably have resulted in her crashing into the tower on the way down). But in the next shot, shown from a distance, she is now about ten feet out and has completely cleared the tower. (00:18:20)

Gavin Jackson

20th Oct 2011

Fast Five (2011)

Question: SPOILERS. We learn that when Vin Diesel and Paul Walker are driving across the bridge, the vault that they are dragging is empty. If this is so, why didn't they both just disconnect and run instead of Vin driving back and causing a huge pile-up? They had the money and dragging an empty vault didn't solve anything.

Gavin Jackson

Chosen answer: Dom wanted Reyes dead, the only way that was going to happen was for the chase to be concluded, and for both Dom and Brian to be caught up by the Police who were in Reyes' pocket. Reyes naturally would think he was safe going near Dom and Brian because of the Police presence.

GalahadFairlight

Answer: Along with putting a stop to Reyes and all the dirty cops, they needed everyone (including Hobbs) to believe that was the real safe, so they could get the actual safe somewhere secure to be able to open it and quietly divvy up and go their own ways.

Stupidity: When Rambo throws the big guy out of the Helicopter and goes after the pilot, all the pilot had to do was tilt the chopper on its side and Rambo would have fallen straight out. Makes more sense than jumping out.

Gavin Jackson

Question: Is there any particular reason as to why Lisbeth is dressed like a punk in the courtroom?

Gavin Jackson

Chosen answer: That is how Lisbeth prefers to dress, and she refuses to change for anyone or anything.

Phixius

29th Jul 2011

Tombstone (1993)

Question: What did Doc mean when he said "It's not revenge he's after. It's the reckoning"?

Gavin Jackson

Chosen answer: A reckoning is like a judgment day, exacting retribution for one's actions. Doc was very well educated and had a very large vocabulary. He was correctly pointing out the subtle difference between revenge (to make Wyatt feel better about losing Morgan and about Virgil's crippling injury) and the fact that Wyatt was bringing about a judgment day (or reckoning) for each of the men who hurt his family.

MovieFan612

Answer: I've spent a lot of time thinking about this very question, and here's what I've come up with. I think there are at least two differences between revenge and a reckoning. First, I think it has to do with the scale of the response to an offending action. Revenge, in my mind, is an eye for an eye, i.e, "You killed my brother and wounded another, so I will inflict the same action on your family (or group, gang, whatever). " A reckoning is less a measured response to an offending action and more of a full-scale punishment, i.e, "You killed my brother and wounded another, so I will now slaughter your entire family-including those who were not directly responsible for the offending action." Second, I think there is also a difference in motivation. Revenge tends to be a very personal response to something, whereas a reckoning tends to be more of a response fueled by a need for justice. In Wyatt's case, it was both. He was enraged by what happened to his family, but was also a lawman.

Franklin Vaughn

Thank you for this response! I've only seen Tombstone a million times and asked the same question every time. It's hard to separate the difference between the two but I believe you nailed it. Well done.

I'm thinking the opposite in terms. Revenge is "Reflexive" and is generally any means necessary (out of an abundance of pain or rage) to hurt the other party. "Revenge is a dish best served cold." If one is exacting justice there's no need to be cold hearted. Therefore, Reckoning is (to me) a fair balancing of the "scales" hence "an eye for an eye." Not only consequences of actions as it were but a corrective action to an incorrect circumstance. Just my understanding.

The problem with that theory is there is no difference in the end because the end result was the same...the killing. True reckoning could have only been achieved though the apprehension and punishment by trial and jury, anything other than that is simply revenge.

16th Jun 2011

X-Men 3 (2006)

Question: I have a few questions. Firstly the latest X-men film showed that although looking only 30ish, Mystique is as old as Professor X. That can only be cause of her mutation. So why after she was cured didn't she age into an older woman? Secondly, if Wolverine had been cured (assuming it would even have worked, given his healing ability), would it have killed him? Given his age and all the adamantium inside him, i couldn't imagine it, but am I wrong?

Gavin Jackson

Chosen answer: Both Wolverine and Mystique's mutations cause them to age slowly; effectively the normal human body deterioration is slowed for them, giving them a younger physical age than their actual chronological age. If that mutation was removed, then the factor that slowed their ageing would no longer function, but they wouldn't abruptly "catch up" with their actual age, they would simply possess a body at their current physical age which would age as a normal human after that point. As for the adamantium lacing Logan's bones, were his healing factor removed, he might well swiftly suffer extreme levels of toxic shock, which would likely prove fatal.

Tailkinker

Question: I've always found it odd that although Wallyworld is closed, the car park is still open and accessible. Is this normal for closed US parks, or could it be considered a goof?

Gavin Jackson

Chosen answer: The parking lot is huge. There's really not any way to close the lot itself, but even if there were gates, Clark would probably have been oblivious to them by then.

Chosen answer: Why would he? It is hardly unusual for a person to never mention a past relationship with someone if they were particularly hurt, angered, or disappointed, resulting in them acting as if that person never existed. It can often come as a surprise to others to learn that a friend or relative had some prior relationship they never knew about.

raywest

8th Jun 2011

Fawlty Towers (1975)

The Anniversary - S2-E5

Question: When Polly is laying in bed disguised as Mrs Fawlty, was that really Prunella scales playing that part? I was wondering as Polly as a more thinner face than Sybil and could never have really got away with it.

Gavin Jackson

Chosen answer: Indeed it is Polly. She has stuffings in her mouth to pretend she is swollen.

Mortug

24th May 2011

Star Wars (1977)

Question: What did Obi-Wan mean when he said to Vader "You can't win, Vader. If you strike me down, I shall become more powerful than you can possibly imagine"? Also when he said "Strike me down", did he specifically mean taking Obi-Wan's life?

Gavin Jackson

Chosen answer: Yes, he meant take Obi-Wan's life. By killing him, Vader would release Obi-Wan from his body allowing him to follow and advise Luke wherever he goes.

Phixius

I believe that Obi-wan would also win because he was willing to "let go" in this situation. Helping Luke was more important than destroying Vader right now.

22nd May 2011

Mafia (1993)

7th Apr 2011

Robocop (1987)

Question: I have two questions about this film. Firstly, was Clarence Boddicker killing cops simply cause he was crazy, or was he doing it under orders from Dick Jones (so he could get his ED-209's online faster)? And secondly, why was Clarence at the drug factory? I always assumed that he was purchasing drugs, but his conversation with the owner didn't mention this. Also it was a lot of money, and he never struck me as that much of a user. Any other reason?

Gavin Jackson

Chosen answer: In part yes. By killing cops along the way, made more of a case for getting the ED-209's brought into service sooner. Clarence was at the drug factory to buy drugs, presumably to sell on for profit and fund his criminal activities.

GalahadFairlight

Answer: Robocop asserts to Sgt. Reed during booking that Boddicker is a "Cop killer" so it's to be assumed he is a ruthless and notorious criminal. Clarence was at the drug factory to form an alliance and partnership with Sal. Clarence has the man power, executive protection and the sales organization to potentially triple Sal's distribution income. Clarence would receive the drugs at wholesale and Clarence would mark up prices.

22nd Feb 2011

Black Moon Rising (1986)

Chosen answer: It was both. Wynn, a fine character actor, was suffering from pancreatic cancer while this film was being made. He died in 1986, the same year the film was released.

raywest

28th Nov 2010

The Expendables (2010)

Other mistake: When Sly Stallone and Jason Statham decide to turn the plane around and attack the dock, they approach it from behind flying above land. But when Statham is firing the guns, one of the shots from his perspective shows it approaching from the water.

Gavin Jackson

5th Oct 2010

Flyboys (2006)

Question: Is it possible to shoot the pilot of an enemy bi-plane in mid-air with a handgun? I would have thought the high wind velocity would have blown the bullet sideways, but I'm hardly an expert.

Gavin Jackson

Chosen answer: Biplanes were not very fast in flight. For instance a Sopwith Camel at about 5000 ft altitude had a cruise speed of approximately 100 mph. A Fokker had a cruise speed of about 95 mph. A typical WWI handgun could shoot 830 feet per second. With the distances between the planes as shown in the movie, a bullet could travel between the planes in less than half a second. The pilot would have to lead the target some, but he conceivably could hit the other pilot if close enough.

rswarrior

5th Oct 2010

Chicken Run (2000)

Question: Why didn't Rocky simply tell Ginger at the start that he couldn't fly, instead of dragging everyone through that whole charade. I realise that it was crucial to the plot, but was there any other reason?

Gavin Jackson

Chosen answer: Because he wanted them all to have something to look forward to, he didn't want to let them down. Plus Rocky is very bigheaded, it would no doubt be embarrassing for him to admit that he couldn't fly.

THGhost

Answer: The exchange was he would teach them to fly so he could hide out from the circus and he didn't have anything else to offer them.

Answer: Because Rocky is a coward.

19th Sep 2010

Short Time (1990)

Other mistake: If Burt honestly believes he only has days left to live, wouldn't he be wondering why he isn't displaying any symptoms of illness (coughing, fever, pain etc). Also why wasn't he prescribed any kind of medication by the doctor who gave him this prognosis. Kinda silly really.

Gavin Jackson

12th Sep 2010

Presumed Innocent (1990)

Continuity mistake: At the end of the film when Harrison Ford is working on the fence, it would have been at least a month since the murder took place. But when he discovers the murder weapon, the blood is still fresh and shows no signs of coagulation.

Gavin Jackson

Question: When this film was being released, I read a rumour somewhere that Lucas originally wanted actor John Houseman to play Indy's father. Given that at the time Houseman was in his mid-80s and in poor health (He passed away when the film was in production), was there any truth to this story?

Gavin Jackson

Chosen answer: Not at least according to the "making of" DVD included in the trilogy box set. According to Spielberg, he always wanted to direct a James Bond film. In Spielberg's thinking, a 1930s action hero could only have one father - James Bond, and also according to Spielberg, he thought only of Connery.

16th Jul 2010

I Am Legend (2007)

Question: Is the scene where Will smith recites all the movie dialogue while it is playing in the original Richard Matheson novel, or was it just a homage to The Omega man (where Charlton Heston does something similar)?

Gavin Jackson

Chosen answer: It's an homage to the Omega Man.

Grumpy Scot

25th May 2010

Avatar (2009)

Question: Why doesn't Jake show any kind of emotion when he watches his brother being cremated at the beginning? I realise that the brother was the higher achiever of the two, but to not show as much as a tear for him seems really odd to me.

Gavin Jackson

Chosen answer: He knew his brother was dead, had been dead for some time. It's likely he'd already done his grieving. Also, he's a marine; he's certainly seen a lot of his comrades fall. Death is part of his way of life.

Phixius

7th May 2010

Creepshow 2 (1987)

Question: Not including the animation sequences, why did this film only have 3 live stories (unlike the original which had 5).

Gavin Jackson

Chosen answer: Beacause the ones from the first ones are short, but the ones from this one are long. So they probly cut it to three to save the budget.

tetracore99

Revealing mistake: When Jason is doing a two fingered push-up on the table in the park, a wire can be seen on his back lifting him up. The mistake has been fixed with proper framing in recent home video versions.

Gavin Jackson

4th Mar 2010

S.W.A.T. (2003)

Chosen answer: Because he was offered a lot of money. $100 million is a lot of money to turn down for someone on a policeman's salary.

LorgSkyegon

4th Mar 2010

Mad Max (1979)

Question: Whatever happened to the 2 cops from the beginning that were in the March hare pursuit vehicle? We never see them again after the beginning. Did they die when their car overturned or what?

Gavin Jackson

Chosen answer: Yes, they died. One of the other MFP officers says after the crash that "they're wasted".

11th Feb 2010

Family Ties (1982)

Show generally

Question: Does anyone know which episode it was when Steven decided to grow a beard, or why he decided to grow one? I never watched the show in order and still can't figure out why he suddenly has one in the second half of the show's run.

Gavin Jackson

Chosen answer: Steven Keaton (Michael Gross) did not grow the beard for an episode. He grew it between season 1 and season 2 as a character he was playing in an Off-Broadway play. When the show began taping for season 2, it was decided to have him keep it. It never was a plot point on the show.

jairodrigue

11th Feb 2010

Cars (2006)

Chosen answer: Nope, nothing official was ever said. There are very few truly original film storylines these days; a close examination of most films would allow one to swiftly note plot similarities to any one of a number of preceding movies. Both Cars and Doc Hollywood fit neatly in a very standard storyline of an outsider coming into an unfamiliar cultural group, which they learn to appreciate and benefit from, while their presence likewise benefits the group they encounter. While the two are unusually similar, no comment on the subject was ever made.

Tailkinker

28th Jan 2010

War of the Worlds (2005)

Question: Does anyone have any clue as to why the aliens are vapourising people when they first appear? They never seem to do it again after that one scene (instead opting to grab them and drain them). Any help would be appreciated.

Gavin Jackson

Chosen answer: The aliens saw the humans as an immediate threat to their ship getting out from underground, so opted to vaporise anyone in the near vicinity so that it could leave the ground unimpeded.

GalahadFairlight

Answer: In the movie there are 3 different Tripods 1. Fighting Machine 2. Brute (Seen at the ferry coming out of the water) 3. Harvesters which take the humans.

Answer: My guess is that they are in search of a specific blood type needed for the growth of the vines that is why some people are harvested and other people with "useless" blood types are killed off.

Question: Why did Obi-wan say to Luke "You will go to the Dagobah system" as opposed to "You must go"? Was he simply giving him an order? Seems strange considering Luke was very close to freezing to death when he said it.

Gavin Jackson

Chosen answer: Obi-wan's one with the Force at this point, so, given that the Force has been shown to grant precognitive visions, he may simply have seen that Luke survives and does go to Dagobah and is telling Luke that he will go there as a statement of fact. Or it could just be a bit poorly worded.

Tailkinker

Factual error: One submission stated that when the Cuban hitman is killed with the arrow from Melina's crossbow, it goes in his back when it should have gone in his side. The corrector stated that we have no idea what angle she was firing from. Well, it is clear that she was standing at the left side of the pool (since we get a point of view shot from her) when he's about to dive in, and she shoots from there. She couldn't have got behind him that fast. So there is no way she could have got him squarely in the back.

Gavin Jackson

Question: I actually have two questions about this film. Firstly, does Rambo actually kill anyone in this film? Dennehy doesn't die and he didn't kill Galt so was there anyone else? And secondly when Rambo arrives at the cliff face, why didn't he simply run to his right or left? The police were only coming at him from behind (and even if one of them came from the side, Rambo could have used his skills to get past him). So why did he feel that going down the cliff was his only option?

Gavin Jackson

Chosen answer: No he doesn't kill anyone. He didn't know if he was surrounded or not and if he did encounter one from the side they might have shot him.

Grumpy Scot

25th Nov 2009

The Running Man (1987)

Plot hole: In the unedited video footage of the helicopter incident shown to the crowd, the last shot of Arnie getting knocked out is seen from his perspective and as such could never have been filmed by any camera.

Gavin Jackson

9th Nov 2009

Casino Royale (2006)

Question: In this film Bond almost dies when he is poisoned and discovers that his portable defibrillator has a broken wire. My question is, has this been the only time in the entire bond series that a Bond gadget has had a fault and not worked properly, or have there been been other instances?

Gavin Jackson

Answer: The defibrillator did not have a fault, due to the fact the Bond nearly falls unconscious he forgot to properly connect both cables to the main device.

Chosen answer: It isn't the first time a Bond gadget didn't work. Most notably, the machine guns mounted on a gyrocopter in You Only Live Twice, and a fake nipple (can't believe I just said that!). It should be noted though that this Bond gadget didn't have a fault, Bond just did not plug the two patches into the Defibrillator, which is what Vesper does when she runs up to the car, thus saving Bond's life.

Answer: In Never Say Never Again, Bond's explosive projectile-shooting pen fires at Fatima Blush, but the projectile that strikes her only explodes after an unexpected delay, leading both her and Bond to assume that it has malfunctioned.

9th Nov 2009

Backdraft (1991)

Chosen answer: Why not? There's no reason why an actor couldn't play his own father at the same age - indeed, it can serve to emphasise a family resemblence between father and son. It can also serve as a swift way of letting the audience know that this is the father, rather than having a different actor come in and have to have the relationship established through dialogue.

Tailkinker

20th Oct 2009

Mad Max 2 (1981)

Question: Where exactly are Humungous and his men getting their gas from? Given the amount of vehicles they have, it would take a fair amount of fuel to run them. But there is no explanation as to where they are getting it.

Gavin Jackson

Chosen answer: The same way Max did until finding the fortress- scavenging, and when possible, attacking other vehicles like they attacked Max at the start of the film. They take what they can find.

johnrosa

Answer: The answer could be referenced to the first movie "Mad Max." In that film it shows some members of the outlaw gang, though not necessarily the ones in "Road Warrior" stealing gas from a tanker by jumping onto the back of it and filling cans; apparently the driver of the tanker unaware. "Road Warrior" was a continuation of "Mad Max."

michael g

13th Oct 2009

Halloween (1978)

Question: I have wondered this for ages. Laurie went over to the Wallace house because she thought they were all playing a prank on her. So when she found the bodies in the bedroom, why did she never assume that this could all just be a practical joke set-up? How did she know straight away that it was the real thing?

Gavin Jackson

Chosen answer: Because it was too graphic and elaborate for her to think it might be a prank. Not to mention that there was probably a very distinctive smell to the room.

JC Fernandez

7th Sep 2009

General questions

I am trying to find the name of a TV movie that I saw back in the 80's. It featured Ken Berry and Sean Marshall (who starred in Pete's Dragon). In the film, Ken's character sees Sean fishing on a beach and they strike up a friendship. Sean lives with his brother and divorced mum who works in a bank. Ken eventually becomes close to the family. At the end whilst in the bank, Sean's little brother dares him to step into the open safe and as a joke promptly closes it, locking him in. They are told that the safe is on a timelock and can't be reopened until the next morning. With no other option, Ken Berry attempts to open it with some tools and is successful. I remember that Max Showalter played the bank manager. I have been unable to find anything on IMDB that links all 3 actors but I know that the film exists. Has anyone else seen this film and knows which one I am talking about?

Gavin Jackson

Chosen answer: It's Valentine's Second Chance (1977). It was an ABC Short Story Special. It's based off the O. Henry story "A Retrieved Reformation. See http://www.hollywood.com/tv/Valentines_Second_Chance/5194277.

11th Aug 2009

Public Enemies (2009)

Question: I must have missed something, but when Dillinger and his accomplice (can't remember his name) are escaping from the lodge, they both hitch a ride with Baby Face Nelson on the dirt road. Later Purvis and his men run Nelson's car off the road and kill him and another guy. Then we see Dillinger and his wounded accomplice with a car in town getting medical supplies. How's that possible?

Gavin Jackson

Chosen answer: Baby Face Nelson doesn't pick up Dillinger. He picks up the characters played by Stephen Dorf and another associate. Dillinger and Hamilton escape through the woods and steal a car from an older man they come across in the forest. After the shootout, Dillinger and Nelson are never in the car together.

30th Jun 2009

Bullitt (1968)

Question: Was the scene of a naked Wolverine arriving at the elderly couple's farmhouse supposed to be some kind of a reference to Superman? The couple seemed to resemble Ma and Pa Kent, the house and farm looked identical to the ones in the Superman films and Logan turning up naked seemed to mirror the fact that baby Clark was naked when he first arrived. So was it intentional?

Gavin Jackson

Chosen answer: There is nothing that the filmakers have said to indicate that this is a reference to Superman. In addition, since this is a Marvel comic, and Superman was was/is a DC comic, it's unlikely.

wizard_of_gore

Question: Was Bryce Dallas Howard actually pregnant when this film was made, or was she just wearing some kind of a cushion under her shirt? Just asking cause no one in the film seems to mention it.

Gavin Jackson

Chosen answer: She wasn't pregnant, no. Her pregnancy, while unremarked upon in this film, will presumably assume greater significance in subsequent films.

Tailkinker

Question: Could someone please explain to me exactly how the Millennium Falcon is being flown. It appears to swoop in such a way like someone is steering it, but no one on board seems to be doing much more than pressing the odd button here or there. And it can't be autopilot because it seems to be doing exactly what those on board want. So does anyone know?

Gavin Jackson

Chosen answer: Han has the control board in front of him - he has everything he needs to fly the ship to hand. We never get a particularly good look at the console, so we don't know precisely what format the controls are presented in, but that's what he's using.

Tailkinker

19th May 2009

Maximum Overdrive (1986)

Question: One of the early posters of this film shows a bearded guy (who is not in the film) coming through a wall crack and holding puppet strings with one hand. Who is this guy supposed to be and what does he represent?

Gavin Jackson

Chosen answer: He does bear a striking resemblance to Stephen King. King was both the writer and director of this movie, and as such, was certainly the guy in charge of all the character's fates and pulling all the strings.

Twotall

Answer: It is Stephen King.

Question: What did the cafe server mean when he said to Marty "I can't give you a tab unless you buy something"? I know that Marty was referring to the Tab soda (which didn't exist then), but what was the other guy talking about?

Gavin Jackson

Chosen answer: A tab is the same as a bill. The server guy thinks Marty wants a bill for whatever he's ordered, although because Marty hasn't eaten or drunk anything yet, he can't give him one. Even though a bill for a restaurant meal can be referred to as a "tab", this term is more commonly used in bars. When someone "runs a tab," it means they pay the total cost as they're about to leave, rather than pay for each drink separately.

raywest

Question: I don't understand why Alex waits until Mrs. Alexander has unchained the opened front door and fully opened it, before he and his droogs break in. I'm sure the four of them could easily have broken the chain off with a bit of force. Is it simply part of Alex's nature to be invited in, before he starts his attack?

Gavin Jackson

Chosen answer: It's part of the "fun element" of the crime to get the victim to open the door themselves.

Captain Defenestrator

The Greatest American Hero (1) - S1-E1

Continuity mistake: In one scene, Ralph is in a phone booth desperately trying to make a phone call. Ralph is nervous and panicky. But when you look at his reflection on the phone, you can see a big relaxed smile on him. When it cuts back to him, he is still acting nervous and panicky.

Gavin Jackson

2nd May 2009

Jurassic Park (1993)

Question: Was there any truth to Hammond's comment that none of the rides at Disneyland worked when the park first opened? I just find that a little hard to believe.

Gavin Jackson

Chosen answer: Yes. The first opening day of Disneyland in California was catastrophic. The pavement was fresh and the sun was so hot high-heeled shoes actually sunk into the walkways. Counterfeit tickets were made, resulting in more people than the park had room for. They ran out of food and drinks. Bathrooms clogged and shut down. Many of the rides broke down on opening day. The Storybook Land Canal Boats had to be pulled by cast members in rubber boots. At the time, there were no guide rails for Autopia; some of the cars crashed into each other, making them inoperable. A gas leak in Fantasyland lead to the land being temporarily closed for part of the day.

David Yard

2nd May 2009

Jurassic Park (1993)

Question: Does anyone know what exactly what the meal was that was served in the conference room? The look on Ellie's face suggests something bad, but does anyone know for sure?

Gavin Jackson

Chosen answer: It was Chilean sea bass; Ellie went a little green around the gills because she had just watched velociraptors tear a cow apart. I don't think I'd want to eat after that.

tattoojunkie

Question: This has baffled me for ages. Instead of tolerating John Bender, why doesn't Richard Vernon simply get him expelled? Vernon must have realised that Bender was not only uncontrollable and a potential threat to him, but also someone who didn't care about his education and was disruptive to others. So why didn't he take the simpler option of getting him out of the school, as opposed to giving him countless detentions?

Gavin Jackson

Chosen answer: Vernon secretly cares about the kids. Pay attention to the scene where he talks to the janitor. He would rather see Bender stay in school and learn something instead of ending up in prison.

23rd Apr 2009

Jurassic Park (1993)

Question: Wouldn't John Hammond be just a little bit worried at how animal rights activists may react to his park feeding live animals (like goats and cows) to the dinosaurs, and the damage it could do to the park's future?

Gavin Jackson

Chosen answer: There are several factors to consider. First, zoos do feed live food to some exhibit animals that will not otherwise eat, like feeding live mice to some types of reptiles. Also, Jurassic Park is still top secret and is not yet open to the public, and therefore Hammond and the staff are, at this point, unconcerned about that and may change their practices later. Another consideration is that the park is in a foreign country that may have less stringent rules and regulations regarding zoo and aquarium practices; Hammond is likely paying them well to establish his park there and is bringing in tourism dollars. Finally, Hammond simply may be unconcerned about it, convinced that his fantastic park will be such a huge success and public demand to see the dinosaurs so great that it will overrule objections by animal rights groups.

raywest

Answer: In Jurassic World they still use animals so this isn't a concern.

What happens in a later movie is irrelevant to the question.

lionhead

13th Apr 2009

Octopussy (1983)

Chosen answer: The satisfaction of knowing he'll win, the thrill of pulling one over on someone; maybe like Goldfinger, he just doesn't like to lose under any circumstances.

Captain Defenestrator

Question: I realise this has been brought up many times, but can it be considered a definite continuity error in the whole series that Obiwan Kenobi is trained by Qui Gon Jinn in this film, despite saying in Empire Strikes Back that it was Yoda who trained him? Some have argued that Qui was working under Yoda, but in Empire, Obiwan said to Yoda, "Was I any different when you taught me?" which to me pretty much says that Yoda was his direct teacher. I'm guessing that Lucas changed the background stories a bit, but I would like to know for sure.

Gavin Jackson

Chosen answer: No, it can't. As we see in Attack of the Clones, Yoda is in charge of training the younglings, the Jedi hopefuls, the ones too young to have been selected as a Padawan. Yoda trained Obi-Wan when he was a child, then Qui-Gon selected him as a Padawan as he reached the proper age. They both trained him, taking responsibility for different stages of his education. It is entirely possible, of course, that Lucas did originally intend Yoda to have been Obi-Wan's sole master when he made The Empire Strikes Back and thus your guess that he changed the background stories may well be accurate, however, if this is the case, then he made the alterations in such a fashion that no continuity error exists, because it all fits together.

Tailkinker

Also, most Jedi probably had friendships with older Jedi who were not their masters. They could think of any Knight/Master as a teacher, despite not being the official apprentice of that Jedi.

8th Apr 2009

Commando (1985)

Question: When Matrix says to his captors "Why not have Bennett do it, looks like something he will get off on"; did he mean it was something Bennett wouldn't go to jail for (considering he was psychotic), or was it some kind of sexual implication?

Gavin Jackson

Chosen answer: A sexual implication, suggesting that it's something Bennett would find exciting.

Tailkinker

Answer: Not sexual but something that he (Bennet) would find immense joy in doing due to his unstable mental nature akin to a sociopathic tendency.

5th Apr 2009

Earthquake (1974)

Question: What was the point of the crazy soldier who shoots the black thieves and later gets shot dead by George Kennedy? Why was he so nutty, and quite frankly what purpose did he even serve in the film?

Gavin Jackson

Chosen answer: Before he put the uniform on, he was a complete nobody, having the respect of no-one. Now that he's in uniform, he demands that people should respect him, and coupled with the powers of Martial Law (where looters can be shot on sight), takes the lack of respect too far and shoots them.

GalahadFairlight

Question: Why does Bond go to Bibi to try and get information about Eric Kriegler? Couldn't he have simply contacted his own department to get a background check on him. After all, it's not like she would have had anything very informative (considering that she had previously stated that he never talks to girls).

Gavin Jackson

Chosen answer: Whether he talked to her or not, Bibi would have inside information simply by being around him so much. She would have overheard things that a background check wouldn't reveal, like planned criminal activity.

21st Mar 2009

Get Smart (1965)

Weekend Vampire - S1-E14

Visible crew/equipment: When Max and 99 have reached the mansion, they both get out of the car to look in the trunk. As they start walking, some lightning occurs. If you look up at the top, you can see where the fake sky ends and the visible back of the stage begins.

Gavin Jackson

21st Mar 2009

Get Smart (1965)

21st Mar 2009

Get Smart (1965)

Washington 4, Indians 3 - S1-E6

Other mistake: When Max tricks the Indian's daughter, he is supposed to say, "Now close your eyes and open your mouth". Instead he says, "Close your mouth and open your eyes". The Indian woman still reacts with closed eyes and open mouth.

Gavin Jackson

23rd Apr 2006

Drop Zone (1994)

Factual error: When Jesse jumps out of Gary Busey's plane near the end, she falls straight down and is able to grab the bar underneath the door. But since she was jumping against high wind pressure.she should have fallen out the door sideways.

Gavin Jackson

Revealing mistake: When Buddy love and Stella are standing at the lookout above LA, the background is obviously a poorly done painting.

Gavin Jackson

20th Mar 2006

X-Men (2000)

Question: Something I never completely understood about the ending. When we see senator Kelly on the TV at the end, we realize that it is actually mystique in disguise. But when the X-men told magneto that Kelly was dead.he didn't believe them. And since Kelly vaporized into water, there would be no body or any proof that he was dead. So wasn't mystique worried that she might come into contact with the real Kelly at some point?

Gavin Jackson

Chosen answer: You are working under the assumption that Mystique would not believe this either. However, with Magneto in prison and no sign of the senator she could happily take on his identity. Also, even if Kelly was alive, after she gave a press statement backing mutants, it would be unlikely that a politician would change his mind so suddenly again and he would therefore not come clean that it was not him.

Scrappy

24th Feb 2006

Falling Down (1993)

Question: Why does the Beggar throw the apple at D-fens? I found it odd since he just gave him a briefcase and a lunch.

Gavin Jackson

Chosen answer: Because the beggar thought there would be items of value in the briefcase and he was frustrated that there was nothing in there. So in frustration he threw the apple at D-Fens.

pierpp

24th Feb 2006

Predator (1987)

Question: Could Hawkins have become a commando (or even a marine) in real life, considering he wore glasses which would have limited him during any battle?

Gavin Jackson

Chosen answer: As long as your uncorrected vision is at least 20/400 (-6.0 diopters), you can qualify for most forms of military service except flying.

Myridon

Answer: Yes, you can have corrected vision using glasses or contacts to be in a special operations unit. Although you can't be color blind.

6th Dec 2005

Superman II (1980)

Question: I don't understand the opening. What was the object that Zod broke in half, and why was it so important (considering it had a guard standing near it)?

Gavin Jackson

Chosen answer: We're never told.

K.C. Sierra

28th Nov 2005

Class of 1984 (1982)

Revealing mistake: At the end when Mr Norris starts to run through the hallways looking for the punks, he is ambushed and beaten up. If you look closely at his shirt, you can see a small pad underneath on his left, the exact spot where they keep punching him.

Gavin Jackson

27th Sep 2005

Phantasm (1979)

Other mistake: When Jody finds the dwarf in the car and recognizes it to be his deceased friend Tommy, he has a quick flashback to Tommy in the coffin and the exact moment Tommy was murdered. But the second flashback makes no sense, Jody never saw Tommy being killed and was told by Reggie that he committed suicide.

Gavin Jackson

27th Sep 2005

Phantasm (1979)

Revealing mistake: When the silver sphere flies into the guy's head, if you look closely, the wire attached to it is visible. This was shot with the ball being pulled away then the footage was reversed.

Gavin Jackson

Question: What exactly happened to Koskov at the end? Pushkin said something like "Put him on a Plane in a diplomatic bag" but was he just being sarcastic or what?

Gavin Jackson

Chosen answer: He says, "Put him on the first plane to Moscow...", making Koskov think he was going to live, but then Pushkin adds, "...in a diplomatic bag." He is being transported back to Russia in a bag...hence, he will be quite dead.

4th Aug 2005

The Karate Kid (1984)

Chosen answer: "San" is a japanese term of respect. Sort of like the english "Mr.", however "Sir" would be a little closer to what it means in japanese. They attach the "San" at the end of someone's name, rather than at the begining like english speakers do.

RJR99SS

22nd Jul 2005

Live and Let Die (1973)

Question: Bond kills Kananga at the end by placing the bullet with the air pellet in his mouth which causes him to inflate and explode. But Bond had the bullet in his own mouth before when he was concealing it. Why didn't it kill him?

Gavin Jackson

Chosen answer: Bond just stored the bullet in his own mouth to hide it. As he puts it in Kananga's mouth he presumably pulls a pin, or forces Kananga to bite it.

Soylent Purple

Chosen answer: Well, we don't know whether Yoda could have defeated Dooku or not, as Dooku resorted to trickery in order to get away - by trying to drop the column onto Anakin and Obi-Wan, then escaping while Yoda's attention was diverted. As such, the battle was inconclusive, although the fact that Dooku had to cheat in order to escape does seem to indicate that he himself doubted his ability to defeat the Jedi Master. As such, we can't really determine whether Anakin or Yoda had the higher power level from their respective battles with Dooku - a more telling indicator would be that Obi-Wan, who is, at best, on level terms with Yoda and is, to be honest, probably less powerful, was ultimately able to defeat Anakin.

Tailkinker

20th Jun 2005

Rocky IV (1985)

Question: In real life, would Drago have faced criminal charges after killing Apollo in the ring? After all he did it on purpose.

Gavin Jackson

Chosen answer: Not really because he only intended on physically punishing Apollo, as do most fighters do when they are in a fight. In real life if Drago had pushed the ref aside like he did in the fight, it would have been stopped right then and there, and maybe Apollo would have lived.

Rollin Garcia Jr

13th Jun 2005

Jaws (1975)

Question: I have been wondering this for ages. In the scene where everyone is on the beach, Ellen Brody sees her husband Chief Brody and waves at him. He waves back. Then she mouths something completely incomprehensible and the chief nods and walks away. What did she say?

Gavin Jackson

Chosen answer: She mouths "I've got Shaun", so he doesn't worry. Alternatively "I've got your order" as she points to the food counter.

Question: I need some translation help here. Firstly when Marion is having the drinking contest and has drunken her final drink, she lowers the glass slowly making everyone think that she has lost. Then she shouts a word twice (Bistoti or something like that) and wins. What does that word mean? Also when Indy is mourning over Marion's "death", two guys in suits come over to him and tell him something in a foreign language. While I have a vague idea of what they said, I was wondering if anyone knew for sure.

Gavin Jackson

Answer: It's Russian, погоди or pogodi. In Russian meaning hold it.

Chosen answer: The two men say to Indy "Der Herr in der Bar mochte immer ihnen sprechen". This is translated as "The man in the bar wants to talk to you". The word in the bar that Marion says is "Pistole", which translated means "Hold it! or "Wait a minute!"

Question: How did Elsa and Donovan get across the second challenge to where the grail was? Indy knew that one had to step on the tiles which spelt Iehovah, but they didn't. I was gonna submit this as a goof, but I am sure there is some logical explanation.

Gavin Jackson

Chosen answer: Dr. Jones senior (Sean Connery) is talking to himself while Indy is going through the traps. He would know what the trials consisted of, and mutters the solutions out loud as he goes over them in his mind. He even makes a statement that "But in ancient Latin, Jehovah begins with an 'I'" just before Indy takes his first step onto the wrong tile.

Twotall

Answer: After Indy falls through the "J" and pulls himself up, you can see Elsa and Donovan standing in the background behind him. It's a very quick shot but they are there. They simply watched him and he didn't see them. Also, Indy called out each letter he stepped on, so they knew the correct path.

Question: I have wondered about this for some time. Did Elsa deliberately choose the wrong grail for Donovan (resulting in his death) cause she knew that Indy would know which one it was and they would both get it without Donovan getting in the way? The smirking expression on her face during the whole scene would definitely imply this but I was wondering if anyone knew for sure?

Gavin Jackson

Chosen answer: Yes, this was a deliberate action on Elsa's part.

Steph_Jared

27th Apr 2005

The Karate Kid (1984)

Question: What exactly was Scotty's reason as to why giving the Company boss the formula for the one inch glass wouldn't alter the future? He gave a brief response, but I honestly can't think of any reason why it wouldn't do any future damage.

Gavin Jackson

Answer: They only give him a schematic of the molecule. The man even says, "It would take years to decipher the matrix", or something like that.

Chosen answer: Scotty says "Why? How do you know he didn't invent the thing!" If the man was in fact the inventor, this would only cause a slight causality loop problem - he "invents" it because they gave it to him, but they only know it because he "invented" it. However, since Sulu said earlier in the movie that it was about 150 years too early for transparent aluminum, it would seem they do know this, so it wasn't a smart thing to do. Of course, the real flaw in the plot is that they need the tank to be transparent at all.

Myridon

Answer: The crew is resigned to the fact that their mission forces them to alter history in some fashion or another. McCoy just wants to acknowledge the gravity of their actions before they go ahead and do it, and Scotty's response is a cheeky way of reassuring him, "Hey, maybe it won't be that bad."

TonyPH

1st Apr 2005

Grease (1978)

Question: I can't understand why Sonny hauled Sandy off the dance floor during the dance finale. Sure Sonny was a bit of a jerk, but he idolized Danny and would never disrespect him. Also there was nothing to suggest that he and Cha-Cha were friends. I never truly understood.

Gavin Jackson

Answer: Watched that scene again now and I just think it was poor writing (in what's still a classic film regardless). How none of the judges noticed that Cha Cha took another girl's (Sandy's) place even after she'd already been "tapped out" is another huge plot hole. Plus a lot of people in America were watching the dance on TV and it's not mentioned by anyone after this scene. 35 years later and that scene still bugs me (haha).

Answer: I agree it plays a bit strange, but I always chalked it up to Sonny wanting Danny to win the dance competition and Sandy, although not bad, admits she's not on Danny's level, while Cha-cha is "the best dancer at St. Bernadette."

Also, he was drinking and upset so not thinking.

Answer: He's fall-down drunk, that's why. He's drinking during the entire dance.

Krista

Answer: It was because they were going to moon the cameras, they didn't want to embarrass Sandy since she would've been offended if it happened when she and Danny were dancing.

Answer: When he's pulling Sandy away he is also yelling to Danny "Go! Go! Go!" So I was also wondering like if Cha-Cha told him to do that and they just failed to add that into the scene.

Chosen answer: It's so Cha-Cha can go with Danny. You can see Cha-Cha giving Danny the eye. So Sonny took Sandy away for Cha-Cha's benefit.

Answer: Because in real life Olivia was late for work, as a result Annette had to take Olivia's place because she was her understudy.

Answer: He is so upset about Marty liking Ed Byrnes, he just drinks all night.

Answer: I think it was because Sonny was drunk and wanted Sandy to talk to Marty for him since she dumped him for Vince Fontaine at the beginning of the dance.

Answer: It's a terrible moment in the story because it exposes Danny as being thoughtless. After already burning Sandy once (at the pep rally) you'd think he'd be mindful of not doing it again. This time, though, he humiliates her in front of the entire school, as well as on television. Unsure how Sandy's sitting with him in the next scene at the drive-in only slightly miffed. Imagine how this plays as a story to their children: "Hey, kids, did I ever tell you about how your dad embarrassed me in front of everybody?" I guess it's meant to be partly motivation for Sandy to change her image and outlook in order to keep Danny, but it's a terrible message (even back then). She's nothing but thoughtful and considerate, and Danny repeatedly treats her badly, but she decides she needs to reinvent herself. (Danny's makeover doesn't count given it lasts about three minutes.) It would've been better had Danny and Sandy came second because of Sandy's dancing, and that could've played on her mind.

Answer: Well my theory is maybe since Sonny wants a girl, Cha-Cha promised him a date with a girl if he agreed to her plan.

Chosen answer: He did. At one point he says amusingly to Starling, "Anthrax Island, that was a nice touch, yours?" indicating he knew the deal wasn't real. That didn't matter to him since he was planning to escape, and the deal gave him an opportunity to do so.

RJR99SS

Answer: The "nice touch" comment came only after Dr. Chilton revealed the fraud. But the part about escaping is very good.

Answer: The anthrax island, nice touch part seems like an in joke, like his anagrams but never worked out what it is?

7th Mar 2005

Stand By Me (1986)

Chosen answer: No, according to later books, Ace never really left Castle Rock and Chris was killed while away at college.

Grumpy Scot

5th Mar 2005

The Grudge (2004)

Other mistake: When Sarah Michelle Gellar is on the bus with her boyfriend, she sees Kayako face looking at her in the window reflection and panics. When she looks back at the window she sees the normal reflection of her face. Trouble is, her reflection is turning her head and looking side to side, whilst Gellar herself is looking straight and not moving her head at all.

Gavin Jackson

9th Feb 2005

Lethal Weapon 4 (1998)

Question: Something I really don't understand about the ending. Riggs is looking over his former wife's grave when his beeper tells him that Lorna is expecting their baby. He races to the hospital and there she is about to give birth. So who drove Lorna to the hospital? Can't have been Riggs as he wouldn't have left at a time of need just to see his wife's grave. Doubtful that Lorna drove herself, and no relatives were there (as we see at the end). So what other explanation is there?

Gavin Jackson

Chosen answer: She could have taken a taxi or an ambulance, or even driven herself. A neighbor or friend could have taken her who didn't stay after she was admitted. She might have already been at the hospital for any number of reasons - doctor's appointment, already admitted for bed rest prior to labor, etc. etc. etc.

Myridon

17th Jan 2005

General questions

Back in the 80's I watched a film on video and I can't remember the title. Set in Italy (I think), its about an bearded inventor who builds a yellow Volkswagen which can do many things like fly, clean up messes, drive sideways, etc., and naturally it can talk. At the end he enters a race pitting him against many people including a pair of nuns. For some reason the Volkswagen continuously betrays him, dropping him from a high distance, leaving him in the middle of nowhere and eventually deciding to let the nuns win. Any help would be much appreciated.

Gavin Jackson

Chosen answer: What you're referring to is SuperBug. A five to seven film series that came out in the 70's. Two or three made it to U.S. theaters. They were made in Germany.

Terminator 2: Judgment Day mistake picture

Other mistake: When the T-1000 walks through the bars at the mental clinic, he gets his handgun stuck in the bars for a few seconds before removing it. In the close-up shot we can see the gun is stuck. But in the wide shot that precedes it, his gun isn't stuck at all. Actor Robert Patrick (T-1000) unconvincingly has his arm wrapped around the bar to make it look like his gun is stuck. (01:02:00)

Gavin Jackson

10th Jan 2005

Canadian Bacon (1995)

Other mistake: Sheriff Bud B. Boomer runs to the top of Toronto's CN tower by staircase and doesn't even draw a breath when he gets there.

Gavin Jackson

4th Jan 2005

Blood Work (2002)

Question: At the end of the film, Is Kiril trying to kill Bourne because it is his mission, or is he doing it simply out of bitterness that he failed the first time?

Gavin Jackson

Chosen answer: It is clear that Gretkov expects Kirill to finish the job he failed the first time around, since Gretkov contacts Kirill after he found out that Bourne was still alive. To quote the movie: Kirill: You told me I had one month off. Gretkov: You told me Jason Bourne was dead.

Andreas[DK]

Also, I question if he was trying to kill Borne. He had a clear shot and hit him in the shoulder. These are trained assassins. No way he doesn't kill Bourne with one shot there. It doesn't make sense that he's a bad shot assassin.

2nd Jan 2005

X-Men 2 (2003)

Question: Why didn't wolverine simply retract his claws when the cops showed up at the front of bobby's house? Might have saved him from being shot.

Gavin Jackson

Chosen answer: They're ordering him to drop them, to put them somewhere where he doesn't have access to them. Obviously he can't do that, however, retracting them isn't going to satisfy them - indeed, if he suddenly retracted them, the shock might actually prompt them to shoot. His best bet is to keep them in view, but make it plain that he's not taking any hostile action with them. Unfortunately, it doesn't work.

Tailkinker

Plot hole: Snowman manages to unload the elephant, summons twenty trucks to the middle of a desert and gets miles ahead of the Bandit for his approach, in what appears to be about ten minutes of time.

Gavin Jackson

16th Dec 2004

X-Men (2000)

Continuity mistake: When Wolverine and Rogue are in Wolverines truck, just prior to the crash, we see them driving near huge trees with thick leaves that can't be seen through. When Rogue says the line "You really should wear your seatbelt", the view out her window shows a handful of trees that one can see through for a mile.

Gavin Jackson

13th Dec 2004

The Frighteners (1996)

Question: How did the people behind the film achieve the effect of making the actors look like ghosts. I truly have no idea.

Gavin Jackson

Chosen answer: For the most part, the actors playing the ghosts were shot separately on blue-screens, and then comped into the scenes. (By removing the blue background through a process called "color keying", and placing them over the scenes) They were simply given a blueish glow and made semi-translucent to complete the effect. It was pretty cutting edge back in the 90's, but nowadays, it'd be very easy to make the exact same effect using a simple program like Adobe After Effects.

TedStixon

27th Nov 2004

The Lost Boys (1987)

Question: When Max approaches the house for his second date with Lucy, being a vampire he asks Michael to invite him in so people in the house would be, in his words, "rendered powerless." As a result, Sam and the Frog brothers can't prove he is a vampire. But how did Max know that his vampirism would be tested in the first place?

Gavin Jackson

Chosen answer: A vampire probably wouldn't survive for very long without playing it safe - he's probably in the habit of insisting on being invited in (if possible) whereever he goes, just in case.

Tailkinker

Answer: He didn't, any vampire cannot enter a home unless they're invited.

27th Nov 2004

Philadelphia (1993)

Question: At the end of the film just before Tom hanks dies, he lowers his oxygen mask and says to partner Banderas "I'm ready". Ready for what exactly? Ready to die? Ready for a bit of final smooching? Or ready (and this is just my assumption) for Banderas to pull the plug?

Gavin Jackson

Chosen answer: Ready to die.

27th Nov 2004

Forever Young (1992)

Question: Mel Gibson spends 50 years frozen inside a machine that his friend Harry built. After Mel awakes, he is still the same age, but he slowly starts to age back into what he would been at the time that he awoke. Later on He discovers that Harry knew this would happen. If Harry knew, why did he build the machine in the first place?. It seems to serve no purpose other than to waste years of someone's life.

Gavin Jackson

Chosen answer: Harry Findlay only discovered the ageing would irreversible after Daniel had been frozen; don't forget this was the first time he'd tried it on a human. Plus he only ever intended to freeze Daniel for a year, so this may not have been the main focus of the experiment anyway.

Answer: He only wanted to be frozen because his girlfriend was in a coma. Harry said, he only tried it on animals and wasn't to keen on the idea. Mel said to wake him when his girlfriend woke up. He figured it would only be for a few weeks or months. Joe Morton, the scientist, read a file on the experiment, stating that Harry died when a fire broke out in the lab and ran in to save him.

26th Oct 2004

Candyman (1992)

Question: I'm confused by the ending. Did Helen herself become just like Candyman, was it really Candyman using Helen's body, or did Helen simply decide to make a brief return from the dead to make her husband pay for his betrayal?

Gavin Jackson

Chosen answer: It's a bit ambiguous, but just as Candyman became a tortured soul who suffered a painful and unjust death, to too did Helen.

18th Sep 2004

Miracle (2004)

Question: Why is this film going straight to video and DVD in Australia?. If it got poor reviews I would understand. But it got good reviews as did Kurt Russell for his performance. I just don't understand.

Gavin Jackson

Chosen answer: The film's international appeal would be very limited to those who knew the story or knew of the event, and this story is really only known well in the US and not in countries like Australia. On the review section, it's not the distributors main priority. Unless they see potential overseas it's costly to try and promote it in Australian cinemas and is likely to see a better audience in video and DVD than theatres.

Lummie

2nd Sep 2004

The Lost Boys (1987)

Question: How is David able to make Michael see the maggots and worms in the Chinese food, considering Michael hasn't yet drunk his blood at this point?

Gavin Jackson

Chosen answer: Many legends give the vampire certain rather hypnotic abilities - this is presumably along the same lines.

Tailkinker

Answer: I always assumed it was because the joint they passed was dusted.

27th Aug 2004

Duel (1971)

Visible crew/equipment: Just after the truck has run down the phone booth at the Reptile lady's place, it starts smashing into all the glass reptile cages nearby. If you look closely at the reptile cage which David falls over, you can see the film crew in the reflection. (00:58:05)

Gavin Jackson

15th Aug 2004

The Time Machine (1960)

Plot hole: In 1966 when George is about to return to his machine and the atomic bombs (or whatever) go off, George just barely gets out of there before the lava from a nearby volcano cooks him alive. Two plot holes here. One is that in the short time it takes George to walk a few meters to his machine, a volcano explodes and the lava reaches him. Thats very speedy lava..especially considering there were no mountains near George's house. Secondly when the lava approaches, it is just flowing along the ground like normal. But when it reaches George, it splashes around him like a broken water tank. That lava can certainly defy gravity.

Gavin Jackson

Question: All of Isaac's followers in this film are young people, no older than about their mid 20's. So why in the opening scene during the Cafe slaughter, is there a elderly lady helping them do their dirty work (the worker who poisons the coffees). This makes no sense at all considering it's the older people they were killing.

Gavin Jackson

Chosen answer: Who knows? Maybe they blackmailed her, because she works there, and the children wouldn't get caught poisoning the coffee, or maybe she isn't as old as you think, she could well be in her mid 20's, it's hard to tell, you only see her briefly.

Hamster

She is obviously in her late teens, early 20s.

31st Jul 2004

The Specialist (1994)

Deliberate mistake: Eric Roberts certainly knows the secret of eternal youth. In the 30 years since his character killed Sharon Stones' parents to the present day, he doesn't appear to have aged a single day.

Gavin Jackson

28th Jul 2004

The Omega Man (1971)

Other mistake: In the opening scene where Charlton Heston drives into a Deserted Los Angeles, he drives past at least 3 people walking down the sidewalk. The first person he passes is quite visibly holding up the traffic for the shooting. Another can be seen on the left of the screen when Heston puts the tape on. And one more can be seen in the wide shot of the street. Other than this, the film makers did a great of making LA look deserted.

Gavin Jackson

25th Jul 2004

Hot Shots! (1991)

22nd Jul 2004

The Karate Kid (1984)

Chosen answer: The Park is called "Golf 'n' Stuff" (like in the movie) and is located in Norwalk, CA. The slide unfortunately was removed in the early 90s.

20th Jul 2004

The Shining (1980)

Question: We see Jack in the picture at the end of the film which was painted in 1921 which I guess means that Nicholson was either possessed or reincarnated...but does anyone have a definite answer? Or does Kubrick just want us to decide for ourselves?

Gavin Jackson

Chosen answer: The idea is that Nicholson's soul has been around for a long time, and that each time he is reborn he returns to the Overlook Hotel and goes on a murder spree. This is facilitated by the evil, restless spirits residing in the hotel. The movie definitely drops some hints that Jack is a reincarnation in the following scenes: After they have been at the Overlook awhile, Wendy talks to Jack upstairs in the bedroom after she brings him breakfast. Jack tells Wendy that he feels like he has been at the Overlook before and upon being given the initial tour of the hotel, he almost knew what was around every corner. According to Delbert Grady during the chilling bathroom scene, Jack has always been the caretaker, which most certainly suggests he is a reincarnation.

Phoenix

15th Jul 2004

Blazing Saddles (1974)

Audio problem: When Taggart kicks over the fake building, he turns around and says "it's a fake, we've been suckered in." But his mouth is wide open and doesn't appear to be saying anything at all. (01:21:25)

Gavin Jackson

Trivia: When the camera crew were rigging the bridge to be blown up, the supervisor in charge of the detonation who didn't speak any foreign language thought he had been given the instruction to blow up the bridge and proceded to do it. Trouble was, the camera crew hadn't finished setting everything up and it was not filmed at all. Sergio Leone was (according to Clint Eastwood) the angriest anyone had ever seen him. As a result, the army agreed to rebuild the bridge for free and it hired a detonator who did speak the right language. This time they got the shot.

Gavin Jackson

Chosen answer: Yes. A "googol" is the number 10 raised to the 100th power, or a 1 followed by 100 zeroes. A "googolplex" is an even larger number - 10 raised to the power of a "googol", or represented as 1 followed by a "googol" zeroes.

BGraz

30th Jun 2004

The Punisher (2004)

29th Jun 2004

Robocop (1987)

Robocop mistake picture

Continuity mistake: When the bearded man is robbing the convenience store, he demands that the owner opens the safe. Look at the Lite cans on the safe. When seen from the right, there are four cans. When the owner kneels down to try and open it, there are now three. When we cut back to a view from the right there are again four. Finally when the robber is shooting at Robocop there are only two. (00:33:20)

Gavin Jackson

26th Jun 2004

Beverly Hills Cop (1984)

25th Jun 2004

Pearl Harbor (2001)

Chosen answer: Probably in Washington. He rejoined active duty in 1940, was assigned to consult with industry re aircraft manufacture; also a trip to England on special mission to evaluate other countries' air power. He requested a return to flying status but was refused. He then was asked to assess feasablity of a air attack on Japan from carrier based planes, and when he asked to lead the mission his request was accepted.

23rd Jun 2004

Freaky Friday (1976)

Continuity mistake: When Ellen is on the hanglider near the end of the film, her hands keep changing from the top of the handle bars to the bottom in almost every shot.

Gavin Jackson

23rd Jun 2004

The Warriors (1979)

Revealing mistake: When Fox is thrown onto the railroad tracks in the subway, if you look closely at his legs you can tell that he stops in midair, meaning he obviously landed on something much higher than the tracks (boxes maybe).

Gavin Jackson

22nd Jun 2004

Freaky Friday (1976)

Revealing mistake: When Annabelle is skiing at the end (just before she turns into Ellen) we see a shot of her from the back of the boat. Its obvious that she is skiing against a backdrop cause the water is too far down for her to be touching it.

Gavin Jackson

21st Jun 2004

Teen Wolf (1985)

Question: Why doesn't Styles just carry a fake ID when he is trying to purchase the keg of beer? Although they aren't foolproof, its a bit smarter than resorting to bribery or using a fake gun.

Gavin Jackson

Chosen answer: Fake ID's are hard to come by. Its also difficult to make one look realistic by yourself.

Grumpy Scot

Styles is Mr. Popularity in a small town. Even if he did have a fake ID, chances are just about everyone knows who he is.

21st Jun 2004

Iron Eagle (1986)

Continuity mistake: When Doug Masters and all his friends are at the burger joint and Masters has just received a letter saying he has been denied enrolment into the US air force academy, his enemy Knotcher tries to humiliate him by announcing it over the loudspeaker. Masters then runs up to the window and threatens to assault him. Knotcher suggests that a race would be more appropriate to which they both agree. During this whole conversation, Knotcher's right arm is up, next shot it's down, then up, then down again.

Gavin Jackson

15th Jun 2004

Dawn of the Dead (2004)

Chosen answer: Most of the interior scenes were shot in a mall in Burlington, near Toronta.

Xofer

Join the mailing list

Separate from membership, this is to get updates about mistakes in recent releases. Addresses are not passed on to any third party, and are used solely for direct communication from this site. You can unsubscribe at any time.

Check out the mistake & trivia books, on Kindle and in paperback.